Monday, September 2, 2019

Basic Steps to Create a Physical Standby Database in Oracle

Let see the basic steps to be followed to create a physical standby database.

Configuration of my servers -  

OS:                - Sun Solaris
Primary Hostname   - HYD-PROD-CBS-LIVE
Standby Hostname   - JPR-DR-CBS-LIVE
Database Version   - 11.2.0.4
Database Name (Primary and Standby): CBSLIVE
Database Unique Name (Primary):   CBS_PROD
Database Unique Name (Standby):   CBS_STDY 
Listener Port (Primary and Standby) -- We had multiple ports here but for example we are taking 1622
Primary Database Files Location   /data*/oradata/cbslive/
Standby Database Files Location   /data*/oradata/cbslive/

1) Create backup of spfile on production

create pfile='/tmp/initCBS_STDY.ora' from spfile;

2) Set following parameters on Production database.
alter system set log_archive_dest_2 = 'service=CBS_STDY';  #Net Service Name of Standby
alter system set log_archive_dest_state_2 = 'defer' ;
alter system set log_archive_config= 'dg_config=(CBS_PROD,CBS_STDY)'; #Add db_unique_name

3) Transfer the password file to $ORACL_HOME/dbs/ location
cd $ORACL_HOME/dbs/
scp orapwCBS_PROD oracle@JPR-DR-CBS-LIVE:$ORACL_HOME/dbs/orapwCBS_STDY

4) Now it's time to take backup. We have different ways to take backup. If we will get downtime and the size of database is small then,
I] Backup - 
a) Using Cold backup ( User Managed )
shutdown the Primary database -
copy all the datafiles to standby.
scp /data*/oradata/cbslive/* oracle@JPR-DR-CBS-LIVE:/data*/oradata/cbslive/*
startup the Primary database.

b) Using Hot Backup ( User Managed )
Put Primary database in Begin Backup mode -
alter database begin backup;
copy all the datafiles to standby.
scp /data*/oradata/cbslive/* oracle@JPR-DR-CBS-LIVE:/data*/oradata/cbslive/*
End the Begin Backup mode -
alter database end backup;

II] Create Standby controlfile on Primary database -
alter database create standby controlfile as '/tmp/control01.ctl';

5) Transfer file '/tmp/initCBS_STDY.ora' and '/tmp/control01.ctl' to Standby server.

6) On standby server - Create standby spfile.
export ORACLE_SID=CBS_STDY
sqlplus / as sysdba
startup nomount pfile='/tmp/initCBS_STDY.ora';
create spfile from pfile='/tmp/initCBS_STDY.ora';
shut immediate;
startup nomount;
exit

7) Enable DG config parameters on standby database - 
alter system set fal_server = 'CBS_STDY';
alter system set db_unique_name = 'CBS_STDY';
alter system set log_archive_config= 'dg_config=(CBS_PROD,CBS_STDY)';

8) Now restore control file
show parameter control file
Check the location and copy the files to that location. I had controlfiles multiplexed at following 3 location.
cp  /tmp/control01.ctl /data/control/cbslive/control01.ctl
cp  /tmp/control01.ctl /data02/control/cbslive/control02.ctl
cp  /tmp/control01.ctl /data03/control/cbslive/control03.ctl

9) Mount the standby database;
alter database mount;

10) Post checks and Config -
a) Crosscheck control file's datafile locations and Physical datafile location if all are present.
b) Create SRL post on standby database. ( Though we create it on primary as well since they might ask for switchover too )
  select * from v$standby_log;
c) Now enable the destination on production to enable log shipping to standby database
  alter system set log_archive_dest_state_2 = 'enable' scope=both;

11) Once everything is placed, Start the MRP
  alter database recover managed standby database disconnect from session;

How to create Physical standby in Oracle

Creating a Physical standby database in oracle requires certain pre-requite to be fulfilled. And creation will be based on scenario and environment you are in!

Pre-requistes -

1. Determine how much disk space will be required.
2. Create same mount points on DR as that of Primary databases.
3. Additional Backup space will be required on both side ie primary as well as standby.
4. Get port opening done between both the servers Primary and Standby database.(Contact with your Network team and Firewall team for more details)
5. Version of Oracle home and grid home installed on standby database should match with Primary database.
6. Primary database in force logging mode and archivelog mode.
7. Listener is configured and running on both servers - Primary and Standby.
8. The Platform/OS of the Standby Database server is the same as the Primary Site
9. We can have hetrogenous standby database depending upon the combination.



Basic Steps for creating a standby database - Click here


1. Backup the primary database including archives.
2. Making the backup available for the standby creation process.
3. Creating the Parameter file & directories if not available on the standby database server.
4. Prepare RMAN standby script.
5. Execute the RMAN script.
6. Additional steps to enable Data guard

Here are scenarios I came across and have tried it.

How To Create Physical Standby On Normal File System on New Host for ASM Primary
How To Create Physical Standby On ASM file system on New Host for ASM Primary
How To Creating Physical Standby Using RMAN DUPLICATE...FROM ACTIVE DATABASE
How To Create Physical Standby using 12c Feature



Sunday, August 18, 2019

How To Recreate a Physical Standby Control file

Recreate of Physical Standby control file is done by us in many cases. When you want to apply incremental backup, we recreate a new control file and restore the control file on physical standby. 
Recreate a Physical Standby Control file requires down time of standby database. Make sure we have downtime in place. As in some environment, Physical Standby  is used for reporting purpose. 

Pre-caution :-
1. Always check status of database, as we are going to shutdown the database.

2. Always your a different location for creating backup control file.

3. Close all the other sessions whenever you are doing an activity which involves shutdown of database.
4. Always make the commands on a notepad before typing it directly on sql prompt.
5. Make sure file structure at production and standby is same ( recommended) but for reason it is not there then additional steps might be required to perform.

 Here we will cover all ways for How To Recreate a Physical Standby Control file -

1) How To Recreate a Physical Standby Control file on Standalone with file system.
2) How To Recreate a Physical Standby Control file on Standalone with ASM.
3) How To Recreate a Physical Standby Control file on RAC.

How To Recreate a Physical Standby Control file on Standalone with file system

Step 1. Log on to standby database. Check the database status -just to make sure we are connected to right database.

SQL> select name, open_mode, database_role from v$database;
NAME      OPEN_MODE            DATABASE_ROLE
--------- -------------------- ----------------
CBSLIVE   MOUNTED              PHYSICAL STANDBY

Take down all the details of datafile location, redolog location, controlfile location and make sure db is started with SPFILE.

select name from v$datafile;
select member from v$logfile;
show parameter spfile;

Step 2. Stop Managed Recovery Process on the Standby Database - 

SQL> alter database recover managed standby database cancel;

Step 3. Shutdown the Standby Database 

SQL> shutdown immediate;

3. Connect to Primary database using SYS as sysdba log in - And check the status.

SQL> select name, open_mode, database_role from v$database;
NAME      OPEN_MODE            DATABASE_ROLE
--------- -------------------- ----------------
CBSLIVE   READ WRITE           PRIMARY

Step 4. Create a new Physical Standby Controlfile from the Primary - 

SQL> alter database create standby controlfile as '/rmanbkp/controlfile/control_for_standby';

Step 5. Backup all the control files on standby database using cp command.

cp /u01/oradata/control/control01.ctl  /rmanbkp/
cp /u02/oradata/control/control02.ctl  /rmanbkp/
cp /u03/oradata/control/control03.ctl  /rmanbkp/

Step 6. SCP ( Transfer the file server.) /rmanbkp/controlfile/control_for_standby to standby server from primary 

Step 7. On Standby database replace all the controlfiles with newly created standby control file.
We had 3 controlfile in our case.

cp /rmanbkp/control_for_standby /u01/oradata/control/control01.ctl
cp /rmanbkp/control_for_standby /u02/oradata/control/control02.ctl
cp /rmanbkp/control_for_standby /u03/oradata/control/control03.ctl

Step 8. Startup the standby database and mount the standby database

SQL> startup nomount;
SQL> alter database mount standby database;

Step 9. If filesystem at standby differs from that of primary, then make sure 

a) set db_file_name_convert or/and log_file_name_convert init parameters in the standby database. 

or 

b) Update the control file by following commands - ( make sure STANDBY_FILE_MANAGEMENT is  MANUAL when you fire below command ) 

SQL> alter database rename file <file#> to '<new File location and file name>';

After completion of rename for all the files which were at different location, make sure STANDBY_FILE_MANAGEMENT is back to what it was.

10. Start Managed Recovery again:

SQL> alter database recover managed standby database disconnect using current logfile parallel 8;

How To Recreate a Physical Standby Control file on Standalone with ASM.

There is only two differences in steps when you are handling a standalone database with asm 
1 The way you take backup (Step 4)
2 The way you restore the backup (Step 7)
Always make sure your database is started with SPFILE , if it is started with pfile then you need to update the control_files location in pfile.


Step 4. Create a new Physical Standby Controlfile from the Primary -


$ export ORACLE_SID=IMPS
RMAN> backup current controlfile for standby format '\rmanbkp\controlfile_for_standby.bkp';



Step 7. Restore the standby control file on standby database.

RMAN> restore standby controlfile from '\rmanbkp\controlfile_for_standby.bkp';

Starting restore at 10-AUG-19
using target database control file instead of recovery catalog
allocated channel: ORA_DISK_1
channel ORA_DISK_1: sid=155 devtype=DISK

channel ORA_DISK_1: restoring control file
channel ORA_DISK_1: restore complete, elapsed time: 00:00:17
output filename=+DATA_IMPS01/IMPS/controlfile/current.267.531045489
output filename=+DATA_IMPS02/IMPS/controlfile/current.257.661096899
output filename=+DATA_IMPS03/IMPS/controlfile/current.327.222958919
Finished restore at 10-AUG-19

RMAN> exit


How To Recreate a Physical Standby Control file on RAC.

When you have multiple nodes, you need to stop the database on both the nodes. And during restore of controlfile, you need to make sure only one instance is up. Rest all steps are same.

Friday, August 16, 2019

Resize Online Redo log on Production database

Here we are going to discuss about how to resize online redo log on production database. Once we had a recommendation and request of resizing online redo log on production database as we were having performance issue ( log file sync wait event). Following post involves doing same activity on different environment we are in. There is no other way till date except you have to add the group and drop the old groups. That is recreating the redo log.

1) Resize Redo Log  on Standalone file system Database.
2) Resize Redo Log  on Standalone ASM Database.
3) Resize Redo Log  on RAC database.

It is a online activity. Though it is advisable to do it on minimum load on the system. 

Pre-cautions :-
1. Make sure you have enough space available on the Redo log member mount point/drive.
2. Keep all the logs before proceeding with any activity. 
3. Always make the commands on a notepad before typing it directly on sql prompt to avoid raising different issue we will discuss in different post :P 

Resize Online Redo Log on Standalone Production Database.


Step 1 Check the database name, status and role just to make sure you logged in to desired db (if you have multiple database on that server) -


SQL> select name, open_mode, database_role from v$database;
NAME      OPEN_MODE            DATABASE_ROLE
--------- -------------------- ----------------
CBSLIVE   READ WRITE           PRIMARY


Step 2 Now check the size of redo log member -


set linesize 1000
set pagesize 1000
set trim on
set lines 120
col group# format 999
col thread# format 999
col member format a80 wrap
col status format a10
col archived format a10
select lg.group#, lg.thread#,lgf.member,lg.archived,lg.status,(bytes/1024/1024) fsize from v$log lg, v$logfile lgf
where lgf.group# = lg.group# order by 1,2;

You will have a output as shown in following manner - In my environment, we had 5 redo log groups which were multiplexed in two different mount point. /u01 and /u02.



GROUP# THREAD# MEMBER                             ARCHIVED STATUS    MB
------ ------- ---------------------------------- -------- --------- ---
     3       1 /u01/oradata/redologs/redo03a.log  NO       INACTIVE  100
     3       1 /u02/oradata/redologs/redo03b.log  NO       INACTIVE  100
     4       1 /u01/oradata/redologs/redo04a.log  NO       ACTIVE    100
     4       1 /u02/oradata/redologs/redo04b.log  NO       ACTIVE    100
     5       1 /u01/oradata/redologs/redo05a.log  NO       CURRENT   100
     5       1 /u02/oradata/redologs/redo05b.log  NO       CURRENT   100
     6       1 /u01/oradata/redologs/redo06a.log  NO       INACTIVE  100
     6       1 /u02/oradata/redologs/redo06b.log  NO       INACTIVE  100
     7       1 /u01/oradata/redologs/redo07a.log  NO       INACTIVE  100
     7       1 /u02/oradata/redologs/redo07b.log  NO       INACTIVE  100


Step 3 Following is the command to create redo log group. Since we already had 3 to 7 redo group, we can't add a new redolog with same number, Hence redo log group got a new number that is 8. Since that redo log member were multiplexed hence there are two file name mentioned in below location. Please note for illustration purpose I have reduced the size to 10M, you have to keep whatever you want to set.


alter database add logfile group 8 ('/u01/oradata/redologs/redo03a.log','/u02/oradata/redologs/redo03b.log') size 10M;

Post addition always crosscheck with following command,


   SQL> select group#, status from v$log;                                   

      GROUP# STATUS
   --------- ----------------
           1 UNUSED
           2 UNUSED
           3 CURRENT
           4 INACTIVE
           5 INACTIVE
           6 INACTIVE
           7 INACTIVE
           8 UNUSED


Step 4 Now you have to follow step 3 and add same number of online redo log group as we had earlier.Here we had 7 online redo log groups, so I have added 7 more online redo log groups following same step 3. ** repeat as necessary until group 8 is CURRENT **


Step 5 Here if you want to drop the groups you have to make sure it is INACTIVE . You can't drop a online redo log group if it is ACTIVE or CURRENT.

Step 6 To make CURRENT TO ACTIVE and then  ACTIVE --> INACTIVE fire switch logfile command-


alter system switch logfile;

alter system checkpoint



Step 7 Now drop all the old redo log groups once they are INACTIVE.
                            

  SQL> alter database drop logfile group 4;
  SQL> alter database drop logfile group 5;
  SQL> alter database drop logfile group 6;


Now just crosscheck the size if it the one which you were looking for.



select group#, bytes, status from v$log;


      GROUP#     BYTES STATUS
   --------- --------- ----------------
           8  10240    CURRENT
           9  10240    UNUSED
          10  10240    UNUSED       

    . . . .<output trimmed> . . . .




Step 8 Delete the files physically if OMF is not enabled.

rm /u01/oradata/redologs/redo03a.log
rm /u02/oradata/redologs/redo03b.log
rm /u01/oradata/redologs/redo04a.log
rm /u02/oradata/redologs/redo04b.log
rm /u01/oradata/redologs/redo05a.log
rm /u02/oradata/redologs/redo05b.log
rm /u01/oradata/redologs/redo06a.log
rm /u02/oradata/redologs/redo06b.log
rm /u01/oradata/redologs/redo07a.log
rm /u02/oradata/redologs/redo07b.log

Resize Online Redo Log on Standalone ASM Database.


When it comes to asm, oracle recommends to enable OMF (Oracle Manage Files) . Here we are doing it with OMF. Make sure these parameters are set - DB_CREATE_FILE_DEST and DB_CREATE_ONLINE_LOG_DEST_n

Changes in OMF and ASM - 
1) When you are adding a diskgroup, never miss the + before diskgroup name, else you will see the redo getting created in $ORACLE_HOME location.
2)You don't have to mention absolute path, while creating redo log group, instead we will mention diskgroup name. For example - 

alter database add logfile group 8 ('+REDO1_CBS','+REDO2_CBS') size 10M;

Rest all steps will be same till 7 as step 8 will not be involved in OMF.

Resize Online Redo Log on RAC Database.

As you all know RAC database involves multiple threads hence your add command will change reset all process will be same. So while adding Online Redo Log (ORL) you have to use following command -  Only Highlighted one is the difference.

alter database add logfile thread 1 group 8 ('+REDO1_CBS','+REDO2_CBS') size 10M;

For every instance we will add desired online redo log group.

Sunday, August 4, 2019

ORACLE DATABASE - INTERVIEW QUESTIONS

Oracle Concepts and Architecture Database Structures


1. What are the components of the physical database structure of Oracle Database?

ORACLE database is comprised of three types of files. One or more data files, two are more Redo Log files and one or more Control files.

2. What are the components of Logical database structure of the ORACLE database?

Tablespaces and the Database's Schema Objects.

3. What is Tablespace?

A database is divided into Logical Storage Unit called tablespaces. A tablespace is used to grouped related logical structures together.

4. What is SYSTEM tablespace and When is it Created?

Every ORACLE database contains a tablespace named SYSTEM, which is automatically created when the database is created. The SYSTEM tablespace always contains the data dictionary tables for the entire database.

5. Explain the relationship among Database, Tablespace and Data file.

Each database logically divided into one or more tablespaces one or more data files are explicitly created for each tablespace.

6. What is schema?

A schema is a collection of database objects of a User.

7. What are the Schema Objects?

Schema objects are the logical structures that directly refer to the database's data. Schema objects include tables, views, sequences, synonyms, indexes, clusters, database triggers, procedures, functions packages, and database links.

8. Can objects of the same Schema reside in a different tablespace?

Yes.

9. Can a Tablespace hold objects from different Schemes?

Yes.

10. what is Table?

A table is the basic unit of data storage in an ORACLE database. The tables of a database hold all of the user-accessible data. Table data is stored in rows and columns.

11. What is a View?

A view is a virtual table. Every view has a Query attached to it. (The Query is a SELECT statement that identifies the columns and rows of the table(s) the view uses.)

12. Do View contain Data ?

Views do not contain or store data.

13. Can a View based on another View?

Yes.

14. What are the advantages of Views?

Provide an additional level of table security, by restricting access to a predetermined set of rows and columns of a table. Hide data complexity. Simplify commands for the user. Present the data in a different perspective from that of the base table.
Store complex queries.

15. What is the Sequence?

A sequence generates a serial list of unique numbers for numerical columns of a database's tables.

16. What is a Synonym?

A synonym is an alias for a table, view, sequence or program unit.

17. What is the type of Synonyms?

There are two types of Synonyms Private and Public.

18. What are Private Synonyms?

A Private Synonyms can be accessed only by the owner.

19. What are Public Synonyms?

Public synonyms can be accessed by any user on the database.

20. What are synonyms used for?

Synonyms are used to -
Mask the real name and owner of an object.
Provide public access to an object
Provide location transparency for tables, views or program units of a remote database.
Simplify the SQL statements for database users.

21. What is an Index?

An Index is an optional structure associated with a table to have direct access to rows, which can be created to increase the performance of data retrieval. An index can be created on one or more columns of a table.

22. How are Indexes Update?

Indexes are automatically maintained and used by ORACLE. Changes to table data are automatically
incorporated into all relevant indexes.

23. What are Clusters?

Clusters are groups of one or more tables physically stores together to share common columns and are often used together.

24. What is cluster Key?

The related columns of the tables in a cluster are called the Cluster Key.

25. What is Index Cluster?

A Cluster with an index on the Cluster Key.

26. What is a Hash Cluster?

A row is stored in a hash cluster based on the result of applying a hash function to the row's cluster key value. All rows with the same hash key value are stores together on disk.

27. When can Hash Cluster used?

Hash clusters are a better choice when a table is often queried with equality queries. For such queries the specified cluster key value is hashed. The resulting hash key value points directly to the area on disk that stores the specified rows.

28. What is Database Link?

A database link is a named object that describes a "path" from one database to another.

29. What are the types of Database Links?

Private Database Link, Public Database Link & Network Database Link.

30. What is Private Database Link?

A private database link is created on behalf of a specific user. A private database link can be used only when the owner of the link specifies a global object name in a SQL statement or in the definition of the owner's views or procedures.

31. What is Public Database Link?

A public database link is created for the special user group PUBLIC. A public database link can be used when any user in the associated database specifies a global object name in a SQL statement or object definition.

32. What is Network Database link?

A network database link is created and managed by a network domain service. A network database link can be used when any user of any database in the network specifies a global object name in a SQL statement or object definition.

33. What is a Data Block?

ORACLE database's data is stored in data blocks. One data block corresponds to a specific number of bytes of physical database space on the disk.

34. How to define the Data Block size?

A data block size is specified for each ORACLE database when the database is created. Database users and allocated free database space in ORACLE data blocks. Block size is specified in INIT.ORA file and can't be changed later.

35. What is Row Chaining?

In Circumstances, all of the data for a row in a table may not be able to fit in the same data block. When this occurs, the data for the row is stored in a chain of the data block (one or more) reserved for that segment.

36. What is the Extent?

An Extent is a specific number of contiguous data blocks, obtained in a single allocation, used to store a specific type of information.

37. What is a Segment?

A segment is a set of extents allocated for a certain logical structure.

38. What is the different type of Segments?

Data Segment, Index Segment, Rollback Segment and Temporary Segment.

39. What is a Data Segment?

Each Non-clustered table has a data segment. All of the table's data is stored in the extents of its data segment. Each cluster has a data segment. The data of every table in the cluster is stored in the cluster's data segment.

40. What is an Index Segment?

Each Index has an Index segment that stores all of its data.

41. What is Rollback Segment?

A Database contains one or more Rollback Segments to temporarily store "undo" information.

42. What are the uses of Rollback Segment?

Rollback Segments are used: To generate read-consistent database information during database recovery to rollback uncommitted transactions for users.

43. What is a Temporary Segment?

Temporary segments are created by ORACLE when a SQL statement needs a temporary work area to
complete execution. When the statement finishes execution, the temporary segment extents are released to the system for future use.

44. What is a Data File?

Every ORACLE database has one or more physical data files. A database's data files contain all the database data. The data of logical database structures such as tables and indexes is physically stored in the data files allocated for a database.

45. What are the Characteristics of Data Files?

A data file can be associated with only one database. Once created a data file can't change size.
One or more data files form a logical unit of database storage called a tablespace.

46. What is a Redo Log?

The set of Redo Log files for a database is collectively known as the database's redo log.

47. What is the function of Redo Log?

The primary function of the redo log is to record all changes made to data.

48. What is the use of Redo Log Information?

The Information in a redo log file is used only to recover the database from a system or media failure prevents database data from being written to a database's data files.

49. What does a Control file Contain?

A Control file records the physical structure of the database. It contains the following information.
Database Name, Names, and locations of a database's files and redo log files.A timestamp of database creation.

50. What is the use of Control File?

When an instance of an ORACLE database is started, its control file is used to identify the database and redo log files that must be opened for database operation to proceed. It is also used in database recovery.

51. What is a Data Dictionary?

The data dictionary of an ORACLE database is a set of tables and views that are used as a read-only reference about the database. It stores information about both the logical and physical structure of the database, the valid users of an ORACLE database, integrity constraints defined for tables in the database and space allocated for a schema object and how much of it is being used.

52. What is an Integrity Constraint?

An integrity constraint is a declarative way to define a business rule for a column of a table.

53. Can an Integrity Constraint be enforced on a table if some existing table data does not satisfy the constraint?

No.

54. Describe the different type of Integrity Constraints supported by ORACLE?

NOT NULL Constraint - Disallows NULLs in a table's column.
UNIQUE Constraint - Disallows duplicate values in a column or set of columns.
PRIMARY KEY Constraint - Disallows duplicate values and NULLs in a column or set of columns.
FOREIGN KEY Constraint - Require each value in a column or set of columns match a value in a related table's
UNIQUE or PRIMARY KEY.
CHECK Constraint - Disallows values that do not satisfy the logical expression of the constraint.

55. What is the difference between UNIQUE constraint and PRIMARY KEY constraint?

A column defined as UNIQUE can contain Nulls while a column defined as PRIMARY KEY can't contain Nulls.

56. Describe Referential Integrity?

A rule defined on a column (or set of columns) in one table that allows the insert or update of a row only if the value for the column or set of columns (the dependent value) matches a value in a column of a related table (the referenced value). It also specifies the type of data manipulation allowed on referenced data and the action to be performed on dependent data as a result of any action on referenced data.

57. What are the Referential actions supported by FOREIGN KEY integrity constraint?

UPDATE and DELETE Restrict - A referential integrity rule that disallows the update or deletion of referenced data.
DELETE Cascade - When a referenced row is deleted all associated dependent rows are deleted.

58. What is self-referential integrity constraint?

If a foreign key reference a parent key of the same table is called self-referential integrity constraint.

59. What are the Limitations of a CHECK Constraint?

The condition must be a Boolean expression evaluated using the values in the row being inserted or updated and can't contain subqueries, sequence, the SYSDATE, UID, USER or USERENV SQL functions, or the pseudo columns LEVEL or ROWNUM.

60. What is the maximum number of CHECK constraints that can be defined on a column?

No Limit.

SYSTEM ARCHITECTURE :

61. What constitutes an ORACLE Instance?

SGA and ORACLE background processes constitute an ORACLE instance. (or) Combination of memory structure and background process.

62. What is SGA?

The System Global Area (SGA) is a shared memory region allocated by ORACLE that contains data and control information for one ORACLE instance.

63. What are the components of SGA?

Database buffers, Redo Log Buffer the Shared Pool and Cursors.

64. What do Database Buffers contain?

Database buffers store the most recently used blocks of database data. It can also contain modified data that has not yet been permanently written to disk.

65. What do Redo Log Buffers contain?

Redo Log Buffer stores redo entries a log of changes made to the database.

66. What is Shared Pool?

Shared Pool is a portion of the SGA that contains shared memory constructs such as shared SQL areas.

67. What is the Shared SQL Area?

A Shared SQL area is required to process every unique SQL statement submitted to a database and contains information such as the parse tree and execution plan for the corresponding statement.

68. What is the Cursor?

A Cursor is a handle ( a name or pointer) for the memory associated with a specific statement.

69. What is PGA?

Program Global Area (PGA) is a memory buffer that contains data and control information for a server process.

70. What is User Process ?

A user process is created and maintained to execute the software code of an application program. It is a shadow process created automatically to facilitate communication between the user and the server process.

71. What is Server Process ?

Server Process handles requests from a connected user process. A server process is in charge of communicating with the user process and interacting with ORACLE carry out requests of the associated user process.

72. What are the two types of Server Configurations?

Dedicated Server Configuration and Multi-threaded Server Configuration.

73. What is Dedicated Server Configuration?

In a Dedicated Server Configuration, a Server Process handles requests for a Single User Process.

74. What is a Multi-threaded Server Configuration?

In a Multi-threaded Server Configuration, many user processes share a group of the server process.

75. What is a Parallel Server option in ORACLE?

A configuration for loosely coupled systems where multiple instance share a single physical database is called Parallel Server.

76. Name the ORACLE Background Process?

DBWR - Database Writer.
LGWR - Log Writer
CKPT - Check Point
SMON - System Monitor
PMON - Process Monitor
ARCH - Archiver
RECO - Recover
Dnnn - Dispatcher, and
LCKn - Lock
Snnn - Server.

77. What Does DBWR do?

Database writer writes modified blocks from the database buffer cache to the data files.

78. When Does DBWR write to the database?

DBWR writes when more data needs to be read into the SGA and too few database buffers are free. The least recently used data is written to the data files first. DBWR also writes when CheckPoint occurs.

79. What does LGWR do?

Log Writer (LGWR) writes redo log entries generated in the redo log buffer of the SGA to on-line Redo Log File.

80. When does LGWR write to the database?

LGWR writes redo log entries into an on-line redo log file when transactions commit and the log buffer files are full.

81. What is the function of checkpoint(CKPT)?

The Checkpoint (CKPT) process is responsible for signaling DBWR at checkpoints and updating all the data files and control files of the database.

82. What are the functions of SMON?

System Monitor (SMON) performs instance recovery at instance start-up. In a multiple instance system (one that uses the Parallel Server), SMON of one instance can also perform instance recovery for other instance that have failed SMON also cleans up temporary segments that are no longer in use and recovers dead transactions skipped during crash and instance recovery because of file-read or off-line errors. These transactions are eventually recovered by SMON when the tablespace or file is brought back on-line SMON also coalesces free extents within the database to make free space contiguous and easier to allocate.

83. What are the functions of PMON?

Process Monitor (PMON) performs process recovery when a user process fails PMON is responsible for cleaning up the cache and freeing resources that the process was using PMON also checks on dispatcher and server processes and restarts them if they have failed.

84. What is the function of ARCH?

Archiver (ARCH) copies the on-line redo log files to archival storage when they are full. ARCH is active only when a database's redo log is used in ARCHIVE LOG mode.

85. What is the function of RECO?

Recover (RECO) is used to resolve distributed transactions that are pending due to a network or system failure in a distributed database. At timed intervals,the local RECO attempts to connect to remote databases and automatically complete the commit or rollback of the local portion of any pending distributed transactions.

86. What is the function of Dispatcher (Dnnn)?

Dispatcher (Dnnn) process is responsible for routing requests from connected user processes to available shared server processes and returning the responses back to the appropriate user processes.

87. How many Dispatcher Processes are created ?

At least one Dispatcher process is created for every communication protocol in use.

88. What is the function of Lock (LCKn) Process ?

Lock (LCKn) are used for inter-instance locking when the ORACLE Parallel Server option is used.

89. What is the maximum number of Lock Processes used ?

Though a single LCK process is sufficient for most Parallel Server systems
upto Ten Locks (LCK0,....LCK9) are used for inter-instance locking.

DATA ACCESS


90. Define Transaction ?

A Transaction is a logical unit of work that comprises one or more SQL statements executed by a single user.

91. When does a Transaction end ?

When it is committed or Rollbacked.

92. What does COMMIT do ?

COMMIT makes permanent the changes resulting from all SQL statements in the transaction. The changes made
by the SQL statements of a transaction become visible to other user sessions transactions that start only after transaction is committed.

93. What does ROLLBACK do ?

ROLLBACK retracts any of the changes resulting from the SQL statements in the transaction.

94. What is SAVE POINT ?

For long transactions that contain many SQL statements, intermediate markers or savepoints can be
declared which can be used to divide a transaction into smaller parts. This allows the option of later rolling back all work performed from the current point in the transaction to a declared savepoint within the
transaction.

95. What is Read-Only Transaction ?

A Read-Only transaction ensures that the results of each query executed in the transaction are consistant with respect to the same point in time.

96. What is the function of Optimizer ?

The goal of the optimizer is to choose the most efficient way to execute a SQL statement.

97. What is Execution Plan ?

The combinations of the steps the optimizer chooses to execute a statement is called an execution plan.

98. What are the different approaches used by Optimizer in choosing an execution plan?

Rule-based and Cost-based.

99. What are the factors that affect OPTIMIZER in choosing an Optimization approach?

The OPTIMIZER_MODE initialization parameter Statistics in the Data Dictionary the
OPTIMIZER_GOAL parameter of the ALTER SESSION commands hints in the statement.

100. What are the values that can be specified for OPTIMIZER MODE Parameter?

COST and RULE.

101. Will the Optimizer always use COST-based approach if OPTIMIZER_MODE is set to "Cost'?

Presence of statistics in the data dictionary for at least one of the tables accessed by the SQL statements is necessary for the OPTIMIZER to use COST-based approach. Otherwise, OPTIMIZER chooses RULE-based approach.

102. What is the effect of setting the value of OPTIMIZER_MODE to 'RULE'?

This value causes the optimizer to choose the rule_based approach for all SQL statements issued to the
instance regardless of the presence of statistics.

103. What are the values that can be specified for OPTIMIZER_GOAL parameter of the ALTER SESSION Command?

CHOOSE, ALL_ROWS, FIRST_ROWS, and RULE.

104. What is the effect of setting the value "CHOOSE" for OPTIMIZER_GOAL, parameter of the ALTER SESSION Command ?

The Optimizer chooses Cost_based approach and optimizes with the goal of best throughput if statistics for atleast one of the tables accessed by the SQL statement exist in the data dictionary. Otherwise the OPTIMIZER chooses RULE_based approach.

105. What is the effect of setting the value "ALL_ROWS" for OPTIMIZER_GOAL parameter of the ALTER SESSION command ?

This value causes the optimizer to the cost-based approach for all SQL statements in the session regardless of the presence of statistics and to optimize with a goal of best throughput.

106. What is the effect of setting the value 'FIRST_ROWS' for OPTIMIZER_GOAL parameter of the ALTER SESSION command?

This value causes the optimizer to use the cost-based approach for all SQL statements in the session regardless of the presence of statistics and to optimize with a goal of best response time.

107. What is the effect of setting the 'RULE' for OPTIMIER_GOAL parameter of the ALTER SESSION Command?

This value causes the optimizer to choose the rule-based approach for all SQL 0statements in a session regardless of the presence of statistics.

108. What is RULE-based approach to optimization?

Choosing an executing plan based on the access paths available and the ranks of these access paths.

109. What is COST-based approach to optimization?

Considering available access paths and determining the most efficient execution plan based on statistics in the data dictionary for the tables accessed by the statement and their associated clusters and indexes.

PROGRAMMATIC CONSTRUCTS


110. What are the different types of PL/SQL program units that can be defined and stored in the ORACLE database?

Procedures and Functions, Packages and Database Triggers.

111. What is a Procedure ?

A Procedure consist of a set of SQL and PL/SQL statements that are grouped together as a unit to solve a specific problem or perform a set of related tasks.

112. What is difference between Procedures and Functions ?

A Function returns a value to the caller where as a Procedure does not.

113. What is a Package?

A Package is a collection of related procedures, functions, variables and other package constructs together as a unit in the database.

114. What are the advantages of having a Package?

Increased functionality (for example, global package variables can be declared and used by any procedure in the package) and performance (for example all objects of the package are parsed compiled, and loaded into memory once)

115. What is Database Trigger?

A Database Trigger is a procedure (set of SQL and PL/SQL statements) that is automatically executed as a result of an insert in, update to, or delete from a table.

116. What are the uses of Database Trigger?

Database triggers can be used to automatic data generation, audit data modifications, enforce complex Integrity constraints, and customize complex security authorizations.

117. What are the differences between Database Trigger and Integrity constraints?

A declarative integrity constraint is a statement about the database that is always true. A constraint applies to existing data in the table and any statement that manipulates the table. A trigger does not apply to data loaded before the definition of the trigger, therefore, it does not guarantee all data in a table conforms to the rules established by an associated trigger. A trigger can be used to enforce transitional constraints whereas a declarative integrity constraint cannot be used.

DATABASE SECURITY


118. What are the Roles ?

Roles are named groups of related privileges that are granted to users or other roles.

119. What is the use of Roles?

REDUCED GRANTING OF PRIVILEGES - Rather than explicitly granting the same set of privileges to many users a database administrator can grant the privileges for a group of related users granted to a role and then grant only the role to each member of the group.
DYNAMIC PRIVILEGE MANAGEMENT - When the privileges of a group must change, only the privileges of the role need to be modified. The security domains of all users granted the group's role automatically reflect the changes made to the role.
SELECTIVE AVAILABILITY OF PRIVILEGES - The roles granted to a user can be selectively enabled (available for use) or disabled (not available for use). This allows specific control of a user's privileges in any given situation.
APPLICATION AWARENESS - A database application can be designed to automatically enable and
disable selective roles when a user attempts to use the application.

120. How to prevent unauthorized use of privileges granted to a Role?

By creating a Role with a password.

121. What is default tablespace?

The Tablespace to contain schema objects created without specifying a tablespace name.

122. What is Tablespace Quota?

The collective amount of disk space available to the objects in a schema on a particular tablespace.

123. What is a profile?

Each database user is assigned a Profile that specifies limitations on various system resources available to the user.

124. What are the system resources that can be controlled through Profile?

The number of concurrent sessions the user can establish the CPU processing time available to the user's session the CPU processing time available to a single call to ORACLE made by a SQL statement the amount of logical I/O available to the user's session the amount of logical I/O available to a single call to ORACLE made by a SQL statement the allowed amount of idle time for the user's session the allowed amount of connect time for the user's session.

125. What is Auditing ?

Monitoring of user access to aid in the investigation of database use.

126. What are the different Levels of Auditing?

Statement Auditing, Privilege Auditing, and Object Auditing.

127. What is Statement Auditing?

Statement auditing is the auditing of the powerful system privileges without regard to specifically named objects.

128. What is Privilege Auditing?

Privilege auditing is the auditing of the use of powerful system privileges without regard to specifically named objects.

129. What is Object Auditing?

Object auditing is the auditing of accesses to specific schema objects without regard to the user.
DISTRIBUTED PROCESSING AND DISTRIBUTED DATABASES

130. What is Distributed database?

A distributed database is a network of databases managed by multiple database servers that appears to a user as a single logical database. The data of all databases in the distributed database can be simultaneously accessed and modified.

131. What is Two-Phase Commit?

Two-phase commit is a mechanism that guarantees a distributed transaction either commits on all involved nodes or rolls back on all involved nodes to maintain data consistency across the globally distributed database. It has two phases, a Prepare Phase and a Commit Phase.

132. Describe two phases of Two-phase commit?

Prepare phase - The global coordinator (initiating node) ask participants to prepare (to promise to commit or rollback the transaction, even if there is a failure)
Commit - Phase - If all participants respond to the coordinator that they are prepared, the coordinator asks all nodes to commit the transaction, if all participants cannot prepare, the coordinator asks all nodes to roll back the transaction.

133. What is the mechanism provided by ORACLE for table replication?

Snapshots and SNAPSHOT LOGs

134. What is a SNAPSHOT?

Snapshots are read-only copies of a master table located on a remote node which is periodically refreshed to reflect changes made to the master table.

135. What is a SNAPSHOT LOG?

A snapshot log is a table in the master database that is associated with the master table. ORACLE uses a snapshot log to track the rows that have been updated in the master table. Snapshot logs are used in updating the snapshots based on the master table.

136. What is a SQL * NET?

SQL *NET is ORACLE's mechanism for interfacing with the communication protocols used by the networks that facilitate distributed processing and distributed databases. It is used in Clint-Server and Server-Server communications.

DATABASE OPERATION, BACKUP, AND RECOVERY


137. What are the steps involved in Database Startup?

Start an instance, Mount the Database and Open the Database.

138. What are the steps involved in Database Shutdown?

Close the Database, Dismount the Database and Shutdown the Instance.

139. What is Restricted Mode of Instance Startup?

An instance can be started in (or later altered to be in) restricted mode so that when the database is open connections are limited only to those whose user accounts have been granted the RESTRICTED SESSION system privilege.

140. What are the different modes of mounting a Database with the Parallel Server?

Exclusive Mode If the first instance that mounts a database does so in exclusive mode, only that Instance can mount the database. Parallel Mode If the first instance that mounts a database is started in parallel mode, other instances that are started in parallel mode can also mount the database.

141. What is the Full Backup?

A full backup is an operating system backup of all data files, on-line redo log files, and control file that constitute the ORACLE database and the parameter.

142. Can Full Backup be performed when the database is open?

Yes

143. What is a Partial Backup?

A Partial Backup is any operating system backup short of a full backup, taken while the database is open or shut down.

144.WhatisOn-line Redo Log?

The On-line Redo Log is a set of two or more on-line redo files that record all committed changes made to the database. Whenever a transaction is committed, the corresponding redo entries temporarily stores in redo log buffers of the SGA are written to an on-line redo log file by the background process LGWR. The on-line redo log files are used in a cyclical fashion.

145. What is Mirrored on-line Redo Log?

A mirrored on-line redo log consists of copies of on-line redo log files physically located on separate disks, changes made to one member of the group are made to all members.

146. What is Archived Redo Log?

Archived Redo Log consists of Redo Log files that have archived before being reused.

147. What are the advantages of operating a database in ARCHIVELOG mode over operating it in NOARCHIVELOG mode?

Complete database recovery from disk failure is possible only in ARCHIVELOG mode. Online database backup is possible only in ARCHIVELOG mode.

148. What is Log Switch?

The point at which ORACLE ends writing to one online redo log file and begins writing to another is called a log switch.

149. What are the steps involved in Instance Recovery?

Rolling forward to recover data that has not been recorded in data files, yet has been recorded in the on-line redo log, including the contents of rollback segments. Rolling back transactions that have been explicitly rolled back or have not been committed as indicated by the rollback segments regenerated in step. Releasing any resources (locks) held by transactions in process at the time of the failure. Resolving any pending distributed transactions undergoing a two-phase commit at the time of the instance failure.

Introduction

1. What is a Database instance? Explain

A database instance (Server) is a set of memory structure and background processes that access a set of database files.The process can be shared by all users. The memory structure that are used to store most queried data from database. This helps up to improve database performance by decreasing the amount of I/O performed against data file.


2. What is Parallel Server?

Multiple instances accessing the same database (Only In Multi-CPU environments)

3. What is a Schema?

The set of objects owned by user account is called the schema.

4. What is an Index? How it is implemented in Oracle Database?

An index is a database structure used by the server to have direct access of a row in a table. An index is automatically created when a unique of primary key constraint clause is specified in create table comman (Ver7.0)

5. What is clusters?

Group of tables physically stored together because they share common columns and are often used together is called Cluster.

6. What is a cluster Key?

The related columns of the tables are called the cluster key. The cluster key is indexed using a cluster index and its value is stored only once for multiple tables in the cluster.

7. What are the basic element of Base configuration of an oracle Database?

It consists of one or more data files.one or more control files.two or more redo log files.The Database contains multiple users/schemas one or more rollback segments one or more tablespaces Data dictionary tables User objects (table,indexes,views etc.,) The server that access the database consists  of
SGA (Database buffer, Dictionary Cache Buffers, Redo log buffers, Shared SQL pool)
SMON (System MONito)
PMON (Process MONitor)
LGWR (LoG Write)
DBWR (Data Base Write)
ARCH (ARCHiver)
CKPT (Check Point)
RECO
Dispatcher
User Process with associated PGS

8. What is a deadlock? Explain .

Two processes wating to update the rows of a table which are locked by the other process then deadlock arises. In a database environment this will often happen because of not issuing proper row lock commands. Poor design of front-end application may cause this situation and the performance of server will reduce drastically. These locks will be released automatically when a commit/rollback operation performed or any one of this processes being killed externally.

MEMORY MANAGEMENT

9. What is SGA? How it is different from Ver 6.0 and Ver 7.0?

The System Global Area in a Oracle database is the area in memory to facilitates the transfer of information between users. It holds the most recently requested structural information between users. It holds the most recently requested structural information about the database. The structure is Database buffers, Dictionary cache, Redo Log Buffer and Shared SQL pool (ver 7.0 only) area.

10. What is a Shared SQL pool?

The data dictionary cache is stored in an area in SGA called the Shared SQL Pool. This will allow sharing of
parsed SQL statements among concurrent users.

11. What is mean by Program Global Area (PGA)?

It is area in memory that is used by a Single Oracle User Process.

12. What is a data segment?

Data segment are the physical areas within a database block in which the data associated with tables and clusters are stored.

13. What are the factors causing the reparsing of SQL statements in SGA?

Due to insufficient Shared SQL pool size. Monitor the ratio of the reloads takes place while executing SQL statements. If the ratio is greater than 1 then increase the SHARED_POOL_SIZE. LOGICAL & PHYSICAL ARCHITECTURE OF DATABASE.

14. What is Database Buffers?

Database buffers are cache in the SGA used to hold the data blocks that are read from the data segments in the database such as tables, indexes and clusters DB_BLOCK_BUFFERS parameter in INIT.ORA decides the size.

15. What is dictionary cache?

Dictionary cache is information about the databse objects stored in a data dictionary table.

16. What is meant by recursive hints?

Number of times processes repeatedly query the dictionary table is called recursive hints. It is due to the data dictionary cache is too small. By increasing the SHARED_POOL_SIZE parameter we can optimize the size of Data Dictionary Cache.

17. What is meant by redo log buffer?

Change made to entries are written to the on-line redo log files. So that they can be used in roll forward operations during database recoveries. Before writing them into the redo log files, they will first brought to redo log buffers in SGA and LGWR will write into files frequently. LOG_BUFFER parameter will decide the size.

18. How will you swap objects into a different table space for an existing database?

Export the user. Perform import using the command imp system/manager file=export.dmp indexfile=newrite.sql. This will create all definitions into newfile.sql. Drop necessary objects.
Run the script newfile.sql after altering the tablespaces. Import from the backup for the necessary objects.

19. List the Optional Flexible Architecture (OFA) of Oracle database? or How can we organize the tablespaces in Oracle database to have maximum performance?

SYSTEM - Data dictionary tables.
DATA - Standard operational tables.
DATA2- Static tables used for standard operations
INDEXES - Indexes for Standard operational tables.
INDEXES1 - Indexes of static tables used for standard operations.
TOOLS - Tools table.
TOOLS1 - Indexes for tools table.
RBS - Standard Operations Rollback Segments,
RBS1,RBS2 - Additional/Special Rollback segments.
TEMP - Temporary purpose tablespace
TEMP_USER - Temporary tablespace for users.
USERS - User tablespace.

20. How will you force database to use particular rollback segment?

SET TRANSACTION USE ROLLBACK SEGMENT rbs_name.

21. What is meant by free extent?

A free extent is a collection of continuous free blocks in tablespace. When a segment is dropped its extents are reallocated and are marked as free.

22. How free extents are managed in Ver 6.0 and Ver 7.0?

Free extents cannot be merged together in Ver 6.0.Free extents are periodically coalesces with the neighboring free extent in Ver 7.0

23.Which parameter in Storage clause will reduce no. of rows per block?

PCTFREE parameter Row size also reduces no of rows per block.

24. What is the significance of having storage clause?

We can plan the storage for a table as how much initial extents are required, how much can be extended next, how much % should leave free for managing row updations etc.,

25. How does Space allocation table place within a block?

Each block contains entries as follows
Fixed block header
Variable block header
Row Header,row date (multiple rows may exists)
PCTEREE (% of free space for row updation in future)

26. What is the role of PCTFREE parameter is Storage clause?

This is used to reserve certain amount of space in a block for expansion of rows.

27. What is the OPTIMAL parameter?

It is used to set the optimal length of a rollback segment.

28. What is the functionality of SYSTEM table space?

To manage the database level transactions such as modifications of the data dictionary table that record information about the free space usage.

29. How will you create multiple rollback segments in a database?

Create a database which implicitly creates a SYSTEM Rollback Segment in a SYSTEM tablespace.
Create a Second Rollback Segment name R0 in the SYSTEM tablespace.
Make new rollback segment available (After shutdown, modify init.ora file and Start database)
Create other tablespaces (RBS) for rollback segments.
Deactivate Rollback Segment R0 and activate the newly created rollback segments.

30. How the space utilisation takes place within rollback segments?

It will try to fit the transaction in a cyclic fashion to all existing extents. Once it found an extent is in use then it forced to acquire a new extent (No. of extents is based on the optimal size)

31. Why query fails sometimes?

Rollback segment dynamically extent to handle larger transactions entry loads.
A single transaction may wipeout all avaliable free space in the Rollback Segment Tablespace. This prevents other user using Rollback segments.

32. How will you monitor the space allocation?

By quering DBA_SEGMENT table/view.

33. How will you monitor rollback segment status?

Querying the DBA_ROLLBACK_SEGS view
IN USE - Rollback Segment is on-line.
AVAILABLE - Rollback Segment available but not on-line.
OFF-LINE - Rollback Segment off-line
INVALID - Rollback Segment Dropped.
NEEDS RECOVERY - Contains data but need recovery or corupted.
PARTLY AVAILABLE - Contains data from an unresolved transaction involving a distributed database.

34. List the sequence of events when a large transaction that exceeds beyond its optimal value when an entry wraps and causes the rollback segment to expand into another extend.

Transaction Begins.
An entry is made in the RES header for new transactions entry
Transaction acquires blocks in an extent of RBS. The entry attempts to wrap into second extent. None is available, so that the RBS must extent. The RBS checks to see if it is part of its OPTIMAL size.
RBS chooses its oldest inactive segment. Oldest inactive segment is eliminated.
RBS extents The Data dictionary table for space management are updated.
Transaction Completes.

35. How can we plan storage for very large tables?

Limit the number of extents in the table
Separate Table from its indexes.
Allocate Sufficient temporary storage.

36. How will you estimate the space required by a non-clustered tables?

Calculate the total header size
Calculate the available dataspace per data block
Calculate the combined column lengths of the average row
Calculate the total average row size.
Calculate the average number rows that can fit in a block
Calculate the number of blocks and bytes required for the table.
After arriving the calculation, add 10 % additional space to calculate the initial extent size for a working table.

37. It is possible to use raw devices as data files and what is the advantages over file. system files?

Yes. The advantages over file system files.
I/O will be improved because Oracle is bye-passing the kernnel which writing into disk. Disk Corruption will be very less.
38. What is a Control file?
Database's overall physical architecture is maintained in a file called control file. It will be used to maintain
internal consistency and guide recovery operations. Multiple copies of control files are advisable.
39. How to implement the multiple control files for an existing database?
Shutdown the databse
Copy one of the existing control file to new location
Edit Config ora file by adding new control file.name
Restart the database.
40. What is meant by Redo Log file mirrorring? How it can be achieved?
Process of having a copy of redo log files is called mirroring.
This can be achieved by creating group of log files together, so that LGWR will automatically writes them to
all the members of the current on-line redo log group. If any one group fails then database automatically switch
over to next group. It degrades performance.
41. What is advantage of having disk shadowing/ Mirroring?
Shadow set of disks save as a backup in the event of disk failure. In most Operating System if any disk failure
occurs it automatically switchover to place of failed disk.
Improved performance because most OS support volume shadowing can direct file I/O request to use the
shadow set of files instead of the main set of files. This reduces I/O load on the main set of disks.
42. What is use of Rollback Segments In Database?
They allow the database to maintain read consistency between multiple transactions.
43. What is a Rollback segment entry?
It is the set of before image data blocks that contain rows that are modified by a transaction.
Each Rollback Segment entry must be completed within one rollback segment.
A single rollback segment can have multiple rollback segment entries.
44. What is hit ratio?
It is a measure of well the data cache buffer is handling requests for data.
Hit Ratio = (Logical Reads - Physical Reads - Hits Misses)/ Logical Reads.
45. When will be a segment released?
When Segment is dropped.
19
When Shrink (RBS only)
When truncated (TRUNCATE used with drop storage option)
46. What are disadvanteges of having raw devices?
We should depend on export/import utility for backup/recovery (fully reliable)
The tar command cannot be used for physical file backup, instead we can use dd command which is less flexible
and has limited recoveries.
47. List the factors that can affect the accuracy of the estimations?
The space used transaction entries and deleted records does not become free immediately after completion due to
delayed cleanout.
Trailling nulls and length bytes are not stored.
Inserts of, updates to and deletes of rows as well as columns larger than a single datablock, can cause
fragmentation an chained row pieces.
DATABASE SECURITY & ADMINISTRATION
48. What is user Account in Oracle database?
An user account is not a physical structure in Database but it is having important relationship to the objects in
the database and will be having certain privileges.
49. How will you enforce security using stored procedures?
Don't grant user access directly to tables within the application.
Instead grant the ability to access the procedures that access the tables.
When procedure executed it will execute the privilege of procedures owner. Users cannot access tables except via
the procedure.
50. What are the dictionary tables used to monitor a database spaces?
DBA_FREE_SPACE
DBA_SEGMENTS
DBA_DATA_FILES.
51. What are the responsibilities of a Database Administrator?
Installing and upgrading the Oracle Server and application tools.
Allocating system storage and planning future storage requirements for the database system.
Managing primary database structures (tablespaces)
Managing primary objects (table,views,indexes)
Enrolling users and maintaining system security.
Ensuring compliance with Oralce license agreement
Controlling and monitoring user access to the database.
Monitoring and optimising the performance of the database.
Planning for backup and recovery of database information.
Maintain archived data on tape
Backing up and restoring the database.
Contacting Oracle Corporation for technical support.
52. What are the roles and user accounts created automatically with the database?
DBA - role Contains all database system privileges.
SYS user account - The DBA role will be assigned to this account. All of the basetables and views for the
database's dictionary are store in this schema and are manipulated only by ORACLE.
20
SYSTEM user account - It has all the system privileges for the database and additional tables and views that
display administrative information and internal tables and views used by oracle tools are created using this
username.
54. What are the database administrators utilities avaliable?
SQL * DBA - This allows DBA to monitor and control an ORACLE database.
SQL * Loader - It loads data from standard operating system files (Flat files) into ORACLE database tables.
Export (EXP) and Import (imp) utilities allow you to move existing data in ORACLE format to and from
ORACLE database.
55. What are the minimum parameters should exist in the parameter file (init.ora)?
DB NAME - Must set to a text string of no more than 8 characters and it will be stored inside the datafiles,
redo log files and control files and control file while database creation.
DB_DOMAIN - It is string that specifies the network domain where the database is created. The global
database name is identified by setting these parameters (DB_NAME & DB_DOMAIN)
CONTORL FILES - List of control filenames of the database. If name is not mentioned then default name will
be used.
DB_BLOCK_BUFFERS - To determine the no of buffers in the buffer cache in SGA.
PROCESSES - To determine number of operating system processes that can be connected to ORACLE
concurrently. The value should be 5 (background process) and additional 1 for each user.
ROLLBACK_SEGMENTS - List of rollback segments an ORACLE instance acquires at database startup.
Also optionally LICENSE_MAX_SESSIONS,LICENSE_SESSION_WARNING and
LICENSE_MAX_USERS.
56. What is a trace file and how is it created?
Each server and background process can write an associated trace file. When an internal error is detected by a
process or user process, it dumps information about the error to its trace. This can be used for tuning the
database.
57. What are roles? How can we implement roles?
Roles are the easiest way to grant and manage common privileges needed by different groups of database users.
Creating roles and assigning provies to roles.
Assign each role to group of users. This will simplify the job of assigning privileges to individual users.
58. What are the steps to switch a database's archiving mode between NO ARCHIVELOG and ARCHIVELOG
mode?
1. Shutdown the database instance.
2. Backup the databse
3. Perform any operating system specific steps (optional)
4. Start up a new instance and mount but do not open the databse.
5. Switch the databse's archiving mode.
59. How can you enable automatic archiving?
Shut the database
Backup the database
Modify/Include LOG_ARCHIVE_START_TRUE in init.ora file.
Start up the databse.
60. How can we specify the Archived log file name format and destination?
By setting the following values in init.ora file.
21
LOG_ARCHIVE_FORMAT = arch %S/s/T/tarc (%S - Log sequence number and is zero left paded, %s - Log
sequence number not padded. %T - Thread number lef-zero-paded and %t - Thread number not padded). The
file name created is arch 0001 are if %S is used.
LOG_ARCHIVE_DEST = path.
61. What is the use of ANALYZE command?
To perform one of these function on an index,table, or cluster:
- to collect statisties about object used by the optimizer and store them in the data dictionary.
- to delete statistics about the object used by object from the data dictionary.
- to validate the structure of the object.
- to identify migrated and chained rows of the table or cluster.
MANAGING DISTRIBUTED DATABASES.
62. How can we reduce the network traffic?
- Replictaion of data in distributed environment.
- Using snapshots to replicate data.
- Using remote procedure calls.
63. What is snapshots?
Snapshot is an object used to dynamically replicate data between distribute database at specified time intervals.
In ver 7.0 they are read only.
64. What are the various type of snapshots?
Simple and Complex.
65. Differentiate simple and complex, snapshots?
- A simple snapshot is based on a query that does not contains GROUP BY clauses, CONNECT BY clauses,
JOINs, sub-query or snashot of operations.
- A complex snapshots contain atleast any one of the above.
66. What dynamic data replication?
Updating or Inserting records in remote database through database triggers. It may fail if remote database is
having any problem.
67. How can you Enforce Refrencial Integrity in snapshots?
Time the references to occur when master tables are not in use.
Peform the reference the manually immdiately locking the master tables. We can join tables in snopshots by
creating a complex snapshots that will based on the master tables.
68. What are the options available to refresh snapshots?
COMPLETE - Tables are completly regenerated using the snapshot's query and the master tables every
time the snapshot referenced.
FAST - If simple snapshot used then a snapshot log can be used to send the changes to the snapshot tables.
FORCE - Default value. If possible it performs a FAST refresh; Otherwise it will perform a complete refresh.
69. what is snapshot log?
It is a table that maintains a record of modifications to the master table in a snapshot. It is stored in the same
database as master table and is only available for simple snapshots. It should be created before creating
snapshots.
70. When will the data in the snapshot log be used?
We must be able to create a after row trigger on table (i.e., it should be not be already available )
22
After giving table privileges.
We cannot specify snapshot log name because oracle uses the name of the master table in the name of the
database objects that support its snapshot log.
The master table name should be less than or equal to 23 characters.
(The table name created will be MLOGS_tablename, and trigger name will be TLOGS name).
72. What are the benefits of distributed options in databases?
Database on other servers can be updated and those transactions can be grouped together with others in a
logical unit.
Database uses a two phase commit.
MANAGING BACKUP & RECOVERY
73. What are the different methods of backing up oracle database?
- Logical Backups
- Cold Backups
- Hot Backups (Archive log)
74. What is a logical backup?
Logical backup involves reading a set of databse records and writing them into a file. Export utility is used for
taking backup and Import utility is used to recover from backup.
75. What is cold backup? What are the elements of it?
Cold backup is taking backup of all physical files after normal shutdown of database. We need to take.
- All Data files.
- All Control files.
- All on-line redo log files.
- The init.ora file (Optional)
76. What are the different kind of export backups?
Full back - Complete database
Incremental - Only affected tables from last incremental date/full backup date.
Cumulative backup - Only affected table from the last cumulative date/full backup date.
77. What is hot backup and how it can be taken?
Taking backup of archive log files when database is open. For this the ARCHIVELOG mode should be
enabled. The following files need to be backed up.
All data files. All Archive log, redo log files. All control files.
78. What is the use of FILE option in EXP command?
To give the export file name.
79. What is the use of COMPRESS option in EXP command?
Flag to indicate whether export should compress fragmented segments into single extents.
80. What is the use of GRANT option in EXP command?
A flag to indicate whether grants on databse objects will be exported or not. Value is 'Y' or 'N'.
81. What is the use of INDEXES option in EXP command?
A flag to indicate whether indexes on tables will be exported.
82. What is the use of ROWS option in EXP command?
23
Flag to indicate whether table rows should be exported. If 'N' only DDL statements for the databse objects will
be created.
83. What is the use of CONSTRAINTS option in EXP command?
A flag to indicate whether constraints on table need to be exported.
84. What is the use of FULL option in EXP command?
A flag to indicate whether full databse export should be performed.
85. What is the use of OWNER option in EXP command?
List of table accounts should be exported.
86. What is the use of TABLES option in EXP command?
List of tables should be exported.
87. What is the use of RECORD LENGTH option in EXP command?
Record length in bytes.
88. What is the use of INCTYPE option in EXP command?
Type export should be performed COMPLETE,CUMULATIVE,INCREMENTAL.
89. What is the use of RECORD option in EXP command?
For Incremental exports, the flag indirects whether a record will be stores data dictionary tables recording the
export.
90. What is the use of PARFILE option in EXP command?
Name of the parameter file to be passed for export.
91. What is the use of PARFILE option in EXP command?
Name of the parameter file to be passed for export.
92. What is the use of ANALYSE ( Ver 7) option in EXP command?
A flag to indicate whether statistical information about the exported objects should be written to export dump
file.
93. What is the use of CONSISTENT (Ver 7) option in EXP command?
A flag to indicate whether a read consistent version of all the exported objects should be maintained.
94. What is use of LOG (Ver 7) option in EXP command?
The name of the file which log of the export will be written.
95.What is the use of FILE option in IMP command?
The name of the file from which import should be performed.
96. What is the use of SHOW option in IMP command?
A flag to indicate whether file content should be displayed or not.
97. What is the use of IGNORE option in IMP command?
A flag to indicate whether the import should ignore errors encounter when issuing CREATE commands.
98. What is the use of GRANT option in IMP command?
A flag to indicate whether grants on database objects will be imported.
24
99. What is the use of INDEXES option in IMP command?
A flag to indicate whether import should import index on tables or not.
100. What is the use of ROWS option in IMP command?
A flag to indicate whether rows should be imported. If this is set to 'N' then only DDL for database objects will
be exectued.

SQL PLUS STATEMENTS
1. What are the types of SQL Statement?
Data Definition Language : CREATE,ALTER,DROP,TRUNCATE,REVOKE,NO AUDIT & COMMIT.
Data Manipulation Language : INSERT,UPDATE,DELETE,LOCK TABLE,EXPLAIN PLAN & SELECT.
Transactional Control : COMMIT & ROLLBACK
Session Control : ALTERSESSION & SET ROLE
System Control : ALTER SYSTEM.
2. What is a transaction?
Transaction is logical unit between two commits and commit and rollback.
3. What is difference between TRUNCATE & DELETE?
TRUNCATE commits after deleting entire table i.e., can not be rolled back. Database triggers do not fire on
TRUNCATE
DELETE allows the filtered deletion. Deleted records can be rolled back or committed.
Database triggers fire on DELETE.
4. What is a join? Explain the different types of joins?
Join is a query which retrieves related columns or rows from multiple tables.
Self Join - Joining the table with itself.
Equi Join - Joining two tables by equating two common columns.
Non-Equi Join - Joining two tables by equating two common columns.
Outer Join - Joining two tables in such a way that query can also retrive rows that do not have corresponding join
value in the other table.
5. What is the Subquery?
Subquery is a query whose return values are used in filtering conditions of the main query.
6. What is correlated sub-query?
Correlated sub_query is a sub_query which has reference to the main query.
7. Explain Connect by Prior?
Retrives rows in hierarchical order.
e.g. select empno, ename from emp where.
8. Difference between SUBSTR and INSTR?
INSTR (String1,String2(n,(m)),
INSTR returns the position of the mth occurrence of the string 2 in
string1. The search begins from nth position of string1.
SUBSTR (String1 n,m)
SUBSTR returns a character string of size m in string1, starting from nth postion of string1.
9. Explain UNION,MINUS,UNION ALL, INTERSECT?
INTERSECT returns all distinct rows selected by both queries.
MINUS - returns all distinct rows selected by the first query but not by the second.
UNION - returns all distinct rows selected by either query
25
UNION ALL - returns all rows selected by either query,including all duplicates.
10. What is ROWID?
ROWID is a pseudo column attached to each row of a table. It is 18 character long, blockno, rownumber are
the components of ROWID.
11. What is the fastest way of accessing a row in a table?
Using ROWID.
CONSTRAINTS
12. What is an Integrity Constraint?
Integrity constraint is a rule that restricts values to a column in a table.
13. What is Referential Integrity?
Maintaining data integrity through a set of rules that restrict the values of one or more columns of the tables
based on the values of primary key or unique key of the referenced table.
14. What are the usage of SAVEPOINTS?
SAVEPOINTS are used to subdivide a transaction into smaller parts. It enables rolling back part of a
transaction. Maximum of five save points are allowed.
15. What is ON DELETE CASCADE?
When ON DELETE CASCADE is specified ORACLE maintains referential integrity by automatically
removing dependent foreign key values if a referenced primary or unique key value is removed.
16. What are the data types allowed in a table?
CHAR,VARCHAR2,NUMBER,DATE,RAW,LONG and LONG RAW.
17. What is difference between CHAR and VARCHAR2? What is the maximum SIZE allowed for each
type?
CHAR pads blank spaces to the maximum length. VARCHAR2 does not pad blank spaces. For CHAR it is 255
and 2000 for VARCHAR2.
18. How many LONG columns are allowed in a table? Is it possible to use LONG columns in WHERE clause
or ORDER BY?
Only one LONG columns is allowed. It is not possible to use LONG column in WHERE or ORDER BY clause.
19. What are the pre requisites?
I. to modify datatype of a column?
ii. to add a column with NOT NULL constraint?
To Modify the datatype of a column the column must be empty.
to add a column with NOT NULL constrain, the table must be empty.
20. Where the integrity constrints are stored in Data Dictionary?
The integrity constraints are stored in USER_CONSTRAINTS.
21. How will you a activate/deactivate integrity constraints?
The integrity constraints can be enabled or disabled by ALTER TABLE ENABLE constraint/DISABLE
constraint.
22. If an unique key constraint on DATE column is created, will it validate the rows that are inserted with
SYSDATE?
26
It won't, Because SYSDATE format contains time attached with it.
23. What is a database link?
Database Link is a named path through which a remote database can be accessed.
24. How to access the current value and next value from a sequence? Is it possible to access the current value
in a session before accessing next value?
Sequence name CURRVAL, Sequence name NEXTVAL.
It is not possible. Only if you access next value in the session, current value can be accessed.
25. What is CYCLE/NO CYCLE in a Sequence?
CYCLE specifies that the sequence continues to generate values after reaching either maximum or
minimum value. After pan ascending sequence reaches its maximum value, it generates its minimum value.
After a descending sequence reaches its minimum, it generates its maximum.
NO CYCLE specifies that the sequence cannot generate more values after reaching its maximum or minimum
value.
26. What are the advantages of VIEW?
To protect some of the columns of a table from other users.
To hide complexity of a query.
To hide complexity of calculations.
27. Can a view be updated/inserted/deleted? If Yes under what conditions?
A View can be updated/deleted/inserted if it has only one base table if the view is based on columns from one or
more tables then insert, update and delete is not possible.
28.If a View on a single base table is manipulated will the changes be reflected on the base table?
If changes are made to the tables which are base tables of a view will the changes be reference on the view.
FORMS 3.0 BASIC
1.What is an SQL *FORMS?
SQL *forms is 4GL tool for developing and executing; Oracle based interactive application.
2. What is the maximum size of a form?
255 character width and 255 characters Length.
3. Name the two files that are created when you generate the form give the filex extension?
INP (Source File)
FRM (Executable File)
4. How do you control the constraints in forms?
Select the use constraint property is ON Block definition screen.
BLOCK
5. Commited block sometimes refer to a BASE TABLE? True or False.
False.
6. Can we create two blocks with the same name in form 3.0?
No.
27
7. While specifying master/detail relationship between two blocks specifying the join condition is a must?
True or False.
True.
8. What is a Trigger?
A piece of logic that is executed at or triggered by a SQL *forms event.
9. What are the types of TRIGGERS?
1. Navigational Triggers.
2. Transaction Triggers.
10. What are the different types of key triggers?
Function Key
Key-function
Key-others
Key-startup
11. What is the difference between a Function Key Trigger and Key Function Trigger?
Function key triggers are associated with individual SQL*FORMS function keys
You can attach Key function triggers to 10 keys or key sequences that normally do not perform any SQL *
FORMS operations. These keys refered as key F0 through key F9.
12. What does an on-clear-block Trigger fire?
It fires just before SQL * forms the current block.
13. How do you trap the error in forms 3.0?
using On-Message or On-Error triggers.
14. State the order in which these triggers are executed?
POST-FIELD,ON-VALIDATE-FIELD,POST-CHANGE and KEY-NEXTFLD.
KEY-NEXTFLD,POST-CHANGE, ON-VALIDATE-FIELD, POST-FIELD.
15. What is the usuage of an ON-INSERT,ON-DELETE and ON-UPDATE TRIGGERS?
These triggers are executes when inserting,deleting and updating operations are performed and can be used to
change the default function of insert,delete or update respectively.
For Eg, instead of inserting a row in a table an existing row can be updated in the same table.
16. When will ON-VALIDATE-FIELD trigger executed?
It fires when a value in a field has been changed and the field status is changed or new and the key has been
pressed. If the field status is valid then any further change to the value in the field will not fire the on-validatefield trigger.
17. A query fetched 10 records How many times does a PRE-QUERY Trigger and POST-QUERY Trigger will
get executed?
PRE-QUERY fires once.
POST-QUERY fires 10 times.
18. What is the difference between ON-VALIDATE-FIELD trigger and a POST-CHANGE trigger?
When you changes the Existing value to null, the On-validate field trigger will fire post change trigger will not
fire. At the time of execute-query post-chage trigger will fire, on-validate field trigger will not fire.
19. What is the difference between an ON-VALIDATE-FIELD trigger and a trigger?
On-validate-field trigger fires, when the field Validation status New or changed.
Post-field-trigger whenever the control leaving form the field, it will fire.
28
20. What is the difference between a POST-FIELD trigger and a POST-CHANGE trigger?
Post-field trigger fires whenever the control leaving from the filed.
Post-change trigger fires at the time of execute-query procedure invoked or filed validation status changed.
21. When is PRE-QUERY trigger executed?
When Execute-query or count-query Package procedures are invoked.
22. Give the sequence in which triggers fired during insert operations, when the following 3 triggers are defined
at the smae block level?
a. ON-INSERT b. POST-INSERT c. PRE-INSERT
PRE-INSERT,ON-INSERT & POST-INSERT.
23. Can we use GO-BLOCK package in a pre-field trigger?
No.
24. Is a Keystartup trigger fires as result of a operator pressing a key explicitly?
No.
25. How can you execute the user defined triggers in forms 3.0?
Execute_Trigger (trigger-name)
26. When does an on-lock trigger fire?
It will fires whenever SQL * Forms would normally attempt to lock a row.
26. What is Post-Block is a
. a. Navigational Trigger.
b. Key trigger
c. Transaction Trigger.
Navigational Trigger.
27. What is the difference between keystartup and pre-form?
Key-startup trigger fires after successful navigation into a form.
Pre-form trigger fires before enter into the form.
28. What is the difference between keystartup and pre-form?
Key-startup triigger fires after successful navigation into a form.
Pre-form trigger fires before enter into the form.
PACKAGE PROCEDURE & FUNCTION
29. What is a Package Procedure?
A Package proecdure is built in PL/SQL procedure.
30. What are the different types of Package Procedure?
1. Restricted package procedure.
2. Unrestricted package proecdure.
31. What is the difference between restricted and unrestricted package procedure?
Restricted package procedure that affects the basic basic functions of SQL * Forms. It cannot used in all triggers
execpt key triggers.
29
Unrestricted package procedure that does not interfere with the basic functions of SQL * Forms it can be used
in any triggers.
32. Classify the restricted and unrestricted procedure from the following.
a. Call
b. User-Exit
c. Call-Query
d. Up
e. Execute-Query
f. Message
g. Exit-From
h. Post
i. Break
a. Call - unrestricted
b. User Exit - Unrestricted
c. Call_query - Unrestricted
d. Up - Restricted
e. Execute Query - Restricted
f. Message - Restricted
g. Exit_form - Restricted
h. Post - Restricted
i. Break - Unrestricted.
33. Can we use a restricted package procedure in ON-VALIDATE-FIELD Trigger?
No.
34. What SYNCHRONIZE procedure does?
It synchoronizes the terminal screen with the internal state of the form.
35. What are the unrestricted procedures used to change the popup screen position during run time?
Anchor-view
Resize -View
Move-View.
36. What Enter package procedure does?
Enter Validate-data in the current validation unit.
37. What ERASE package procedure does?
Erase removes an indicated global variable.
38. What is the difference between NAME_IN and COPY?
Copy is package procedure and writes values into a field.
Name in is a package function and returns the contents of the variable to which you apply.
38. Identify package function from the following?
1. Error-Code
2. Break
3. Call
4. Error-text
5. Form-failure
6. Form-fatal
30
7. Execute-query
8. Anchor_View
9. Message_code
1. Error_Code
2. Error_Text
3. Form_Failure
4. Form_Fatal
5. Message_Code
40. How does the command POST differs from COMMIT?
Post writes data in the form to the database but does not perform database commit
Commit permenently writes data in the form to the database.
41. What the PAUSE package procedure does?
Pause suspends processing until the operator presses a function key
42. What package procedure is used for calling another form?
Call (E.g. Call(formname)
43. What package procedure used for invoke sql *plus from sql *forms?
Host (E.g. Host (sqlplus))
44. Error_Code is a package proecdure?
a. True b. false
False.
45. EXIT_FORM is a restricted package procedure?
a. True b. False
True.
46. When the form is running in DEBUG mode, If you want to examine the values of global variables and
other form variables, What package procedure command you would use in your trigger text?
Break.
SYSTEM VARIABLES
47. List the system variables related in Block and Field?
1. System.block_status
2. System.current_block
3. System.current_field
4. System.current_value
5. System.cursor_block
6. System.cursor_field
7. System.field_status.
48. What is the difference between system.current_field and system.cursor_field?
1. System.current_field gives name of the field.
2. System.cursor_field gives name of the field with block name.
49. The value recorded in system.last_record variable is of type
31
a. Number
b. Boolean
c. Character.
b. Boolean.
User Exits :
50. What is an User Exits?
A user exit is a subroutine which are written in programming languages using pro*C pro *Cobol , etc., that
link into the SQL * forms executable.
51. What are the type of User Exits?
ORACLE Precompliers user exits
OCI (ORACLE Call Interface)
Non-ORACEL user exits.
Page:
52. What do you mean by a page?
Pages are collection of display information, such as constant text and graphics.
53. How many pages you can in a single form?
Unlimited.
54. Two popup pages can appear on the screen at a time?
a. True b. False
a. True.
55.What is the significance of PAGE 0 in forms 3.0?
Hide the fields for internal calculation.
56. Deleting a page removes information about all the fields in that page?
a. True. b. False
a. True.
Popup Window :
57. What do you mean by a pop-up window?
Pop-up windows are screen areas that overlay all or a portion of the
display screen when a form is running.
58. What are the types of Pop-up window?
the pop-up field editor
pop-up list of values
pop-up pages.
Alert :
59. What is an Alert?
An alert is window that appears in the middle of the screen overlaying a portion of the current display.


FORMS 4.0
01. Give the Types of modules in a form?
Form
32
Menu
Library
02. Write the Abbreviation for the following File Extension
1. FMB 2. MMB 3. PLL
FMB ----- Form Module Binary.
MMB ----- Menu Module Binary.
PLL ------ PL/SQL Library Module Binary.
03. What are the design facilities available in forms 4.0?
Default Block facility.
Layout Editor.
Menu Editor.
Object Lists.
Property Sheets.
PL/SQL Editor.
Tables Columns Browser.
Built-ins Browser.
04. What is a Layout Editor?
The Layout Editor is a graphical design facility for creating and arranging items and boilerplate text
and graphics objects in your application's interface.
BLOCK
05. What do you mean by a block in forms4.0?
Block is a single mechanism for grouping related items into a functional unit for storing,displaying and
manipulating records.
06. Explain types of Block in forms4.0?
Base table Blocks.
Control Blocks.
1. A base table block is one that is associated with a specific database table or view.
2. A control block is a block that is not associated with a database table.
ITEMS
07. List the Types of Items?
Text item.
Chart item.
Check box.
Display item.
Image item.
List item.
Radio Group.
User Area item.
08. What is a Navigable item?
A navigable item is one that operators can navigate to with the keyboard during default navigation, or that
Oracle forms can navigate to by executing a navigational
built-in procedure.
09. Can you change the color of the push button in design time?
33
No.
10. What is a Check Box?
A Check Box is a two state control that indicates whether a certain condition or value is on or off, true or
false. The display state of a check box is always either "checked" or "unchecked".
11. What are the triggers associated with a check box?
Only When-checkbox-activated Trigger associated with a Check box.
PL/SQL
Basiscs of PL/SQL
1. What is PL/SQL?
PL/SQL is a procedural language that has both interactive SQL and procedural programming language
constructs such as iteration, conditional branching.
2. What is the basic structure of PL/SQL?
PL/SQL uses block structure as its basic structure. Anonymous blocks or nested blocks can be used in PL/SQL.
3. What are the components of a PL/SQL block?
A set of related declarations and procedural statements is called block.
4. What are the components of a PL/SQL Block?
Declarative part, Executable part and Execption part.
Datatypes PL/SQL
5. What are the datatypes a available in PL/SQL?
Some scalar data types such as NUMBER, VARCHAR2, DATE, CHAR, LONG, BOOLEAN.
Some composite data types such as RECORD & TABLE.
6. What are % TYPE and % ROWTYPE? What are the advantages of using these over datatypes?
% TYPE provides the data type of a variable or a database column to that variable.
% ROWTYPE provides the record type that represents a entire row of a table or view or columns selected in the
cursor.
The advantages are : I. Need not know about variable's data type
ii. If the database definition of a column in a table changes, the data type of a variable changes accordingly.
7. What is difference between % ROWTYPE and TYPE RECORD?
% ROWTYPE is to be used whenever query returns a entire row of a table or view.
TYPE rec RECORD is to be used whenever query returns columns of different
table or views and variables.
E.g. TYPE r_emp is RECORD (eno emp.empno% type,ename emp ename %type
);
e_rec emp% ROWTYPE
cursor c1 is select empno,deptno from emp;
e_rec c1 %ROWTYPE.
8. What is PL/SQL table?
Objects of type TABLE are called "PL/SQL tables", which are modelled as (but not the same as) database
tables, PL/SQL tables use a primary PL/SQL tables can have one column and a primary key.
34
Cursors
9. What is a cursor? Why Cursor is required?
Cursor is a named private SQL area from where information can be accessed. Cursors are required to process
rows individually for queries returning multiple rows.
10. Explain the two type of Cursors?
There are two types of cursors, Implict Cursor and Explicit Cursor.
PL/SQL uses Implict Cursors for queries.
User defined cursors are called Explicit Cursors. They can be declared and used.
11. What are the PL/SQL Statements used in cursor processing?
DECLARE CURSOR cursor name, OPEN cursor name, FETCH cursor name INTO or Record types,
CLOSE cursor name.
12. What are the cursor attributes used in PL/SQL?
%ISOPEN - to check whether cursor is open or not
% ROWCOUNT - number of rows featched/updated/deleted.
% FOUND - to check whether cursor has fetched any row. True if rows are featched.
% NOT FOUND - to check whether cursor has featched any row. True if no rows are featched.
These attributes are proceded with SQL for Implict Cursors and with Cursor name for Explict Cursors.
13. What is a cursor for loop?
Cursor for loop implicitly declares %ROWTYPE as loop index,opens a cursor, fetches rows of values from
active set into fields in the record and closes
when all the records have been processed.
eg. FOR emp_rec IN C1 LOOP
salary_total := salary_total +emp_rec sal;
END LOOP;
14. What will happen after commit statement?
Cursor C1 is
Select empno,
ename from emp;
Begin
open C1; loop
Fetch C1 into
eno.ename;
Exit When
C1 %notfound;-----
commit;
end loop;
end;
The cursor having query as SELECT .... FOR UPDATE gets closed after COMMIT/ROLLBACK.
The cursor having query as SELECT.... does not get closed even after COMMIT/ROLLBACK.
15. Explain the usage of WHERE CURRENT OF clause in cursors?
WHERE CURRENT OF clause in an UPDATE,DELETE statement refers to the latest row fetched from a cursor.
Database Triggers
16. What is a database trigger? Name some usages of database trigger?
35
Database trigger is stored PL/SQL program unit associated with a specific database table. Usages are Audit
data modificateions, Log events transparently, Enforce complex business rules Derive column values
automatically, Implement complex security authorizations. Maintain replicate tables.
17. How many types of database triggers can be specified on a table? What are they?
Insert Update Delete
Before Row o.k. o.k. o.k.
After Row o.k. o.k. o.k.
Before Statement o.k. o.k. o.k.
After Statement o.k. o.k. o.k.
If FOR EACH ROW clause is specified, then the trigger for each Row affected by the statement.
If WHEN clause is specified, the trigger fires according to the retruned boolean value.
18. Is it possible to use Transaction control Statements such a ROLLBACK or COMMIT in Database Trigger?
Why?
It is not possible. As triggers are defined for each table, if you use COMMIT of ROLLBACK in a trigger,
it affects logical transaction processing.
19. What are two virtual tables available during database trigger execution?
The table columns are referred as OLD.column_name and NEW.column_name.
For triggers related to INSERT only NEW.column_name values only available.
For triggers related to UPDATE only OLD.column_name NEW.column_name values only available.
For triggers related to DELETE only OLD.column_name values only available.
20. What happens if a procedure that updates a column of table X is called in a database trigger of the same table
?
Mutation of table occurs.
21. Write the order of precedence for validation of a column in a table?
I. done using Database triggers.
ii. done using Integarity Constraints.
I & ii.
Exception :
22. What is an Exception? What are types of Exception?
Exception is the error handling part of PL/SQL block. The types are Predefined and user_defined. Some of
Predefined execptions are.
CURSOR_ALREADY_OPEN
DUP_VAL_ON_INDEX
NO_DATA_FOUND
TOO_MANY_ROWS
INVALID_CURSOR
INVALID_NUMBER
LOGON_DENIED
NOT_LOGGED_ON
PROGRAM-ERROR
STORAGE_ERROR
TIMEOUT_ON_RESOURCE
VALUE_ERROR
ZERO_DIVIDE
OTHERS.
36
23. What is Pragma EXECPTION_INIT? Explain the usage?
The PRAGMA EXECPTION_INIT tells the complier to associate an exception with an oracle error. To get an
error message of a specific oracle error.
e.g. PRAGMA EXCEPTION_INIT (exception name, oracle error number)
24. What is Raise_application_error?
Raise_application_error is a procedure of package DBMS_STANDARD which allows to issue an
user_defined error messages from stored sub-program or database trigger.
25. What are the return values of functions SQLCODE and SQLERRM?
SQLCODE returns the latest code of the error that has occured.
SQLERRM returns the relevant error message of the SQLCODE.
26. Where the Pre_defined_exceptions are stored?
In the standard package.
Procedures, Functions & Packages ;
27. What is a stored procedure?
A stored procedure is a sequence of statements that perform specific function.
28. What is difference between a PROCEDURE & FUNCTION?
A FUNCTION is alway returns a value using the return statement.
A PROCEDURE may return one or more values through parameters or may not return at all.
29. What are advantages fo Stored Procedures /
Extensibility,Modularity, Reusability, Maintainability and one time compilation.
30. What are the modes of parameters that can be passed to a procedure?
IN,OUT,IN-OUT parameters.
31. What are the two parts of a procedure?
Procedure Specification and Procedure Body.
32. Give the structure of the procedure?
PROCEDURE name (parameter list.....)
is
local variable declarations
BEGIN
Executable statements.
Exception.
exception handlers
end;
33. Give the structure of the function?
FUNCTION name (argument list .....) Return datatype is
local variable declarations
Begin
executable statements
Exception
execution handlers
End;
37
34. Explain how procedures and functions are called in a PL/SQL block?
Function is called as part of an expression.
sal := calculate_sal ('a822');
procedure is called as a PL/SQL statement
calculate_bonus ('A822');
35. What is Overloading of procedures?
The Same procedure name is repeated with parameters of different datatypes and parameters in different
positions, varying number of parameters is called overloading of procedures.
e.g. DBMS_OUTPUT put_line
36. What is a package? What are the advantages of packages?
Package is a database object that groups logically related procedures.
The advantages of packages are Modularity, Easier Applicaton Design, Information. Hiding,. reusability and
Better Performance.
37.What are two parts of package?
The two parts of package are PACKAGE SPECIFICATION & PACKAGE BODY.
Package Specification contains declarations that are global to the packages and local to the schema.
Package Body contains actual procedures and local declaration of the procedures and cursor declarations.
38. What is difference between a Cursor declared in a procedure and Cursor declared in a package specification?
A cursor declared in a package specification is global and can be accessed by other procedures or procedures in a
package.
A cursor declared in a procedure is local to the procedure that can not be accessed by other procedures.
39. How packaged procedures and functions are called from the following?
a. Stored procedure or anonymous block
b. an application program such a PRC *C, PRO* COBOL
c. SQL *PLUS
a. PACKAGE NAME.PROCEDURE NAME (parameters);
variable := PACKAGE NAME.FUNCTION NAME (arguments);
EXEC SQL EXECUTE
b.
BEGIN
PACKAGE NAME.PROCEDURE NAME (parameters)
variable := PACKAGE NAME.FUNCTION NAME (arguments);
END;
END EXEC;
c. EXECUTE PACKAGE NAME.PROCEDURE if the procedures does not have any
out/in-out parameters. A function can not be called.
40. Name the tables where characteristics of Package, procedure and functions are stored?
User_objects, User_Source and User_error.
FORMS4.0
12. what is a display item?
Display items are similar to text items but store only fetched or assigned values. Operators cannot navigate to a
display item or edit the value it contains.
13. What is a list item?
38
It is a list of text elements.
14. What are the display styles of list items?
Poplist, No text Item displayed in the list item.
Tlist, No element in the list is highlighted.
15. What is a radio Group?
Radio groups display a fixed no of options that are mutually Exclusive .
User can select one out of n number of options.
16. How many maximum number of radio buttons can you assign to a radio group?
Unlimited no of radio buttons can be assigned to a radio group
17. can you change the default value of the radio button group at run time?
No.
18.What triggers are associated with the radio group?
Only when-radio-changed trigger associated with radio group
Visual Attributes.
19. What is a visual attribute?
Visual Attributes are the font, color and pattern characteristics of objects that operators see and intract with
in our application.
20. What are the types of visual attribute settings?
Custom Visual attributes
Default visual attributes
Named Visual attributes.
Window
21. What is a window?
A window, byitself , can be thought of as an empty frame. The frame provides a way to intract with the
window, including the ability to scroll, move, and resize the window. The content of the window ie. what is
displayed inside the frame is determined by the canvas View or canvas-views displayed in the window at runtime.
22. What are the differrent types of windows?
Root window, secondary window.
23. Can a root window be made modal?
No.
24. List the buil-in routine for controlling window during run-time?
Find_window,
get_window_property,
hide_window,
move_window,
resize_window,
set_window_property,
show_View
39
25. List the windows event triggers available in Forms 4.0?
When-window-activated, when-window-closed, when-window-deactivated,
when-window-resized
26. What built-in is used for changing the properties of the window dynamically?
Set_window_property
Canvas-View
27. What is a canvas-view?
A canvas-view is the background object on which you layout the interface items (text-items, check boxes,
radio groups, and so on.) and boilerplate objects that operators see and interact with as they run your form. At
run-time, operators can see only those items that have been assiged to a specific canvas. Each canvas, in
term, must be displayed in a specfic window.
28. Give the equivalent term in forms 4.0 for the following.
Page, Page 0?
Page - Canvas-View
Page 0 - Canvas-view null.
29. What are the types of canvas-views?
Content View, Stacked View.
30. What is the content view and stacked view?
A content view is the "Base" view that occupies the entire content pane of the window in which it is displayed.
A stacked view differs from a content canvas view in that it is not the base view for the window to which it is
assigned
31. List the built-in routines for the controlling canvas views during run-time?
Find_canvas
Get-Canvas_property
Get_view_property
Hide_View
Replace_content_view
Scroll_view
Set_canvas_property
Set_view_property
Show_view
Alert
32. What is an Alert?
An alert is a modal window that displays a message notifies the operator of some application condition
33. What are the display styles of an alert?
Stop, Caution, note
34. Can you attach an alert to a field?
No
35. What built-in is used for showing the alert during run-time?
Show_alert.
40
36. Can you change the alert messages at run-time?
If yes, give the name of th built-in to chage the alert messages at run-time.
Yes. Set_alert_property.
37. What is the built-in function used for finding the alert?
Find_alert
Editors
38. List the editors availables in forms 4.0?
Default editor
User_defined editors
system editors.
39. What buil-in routines are used to display editor dynamicaly?
Edit_text item
show_editor
LOV
40. What is an Lov?
A list of values is a single or multi column selection list displayed in
a pop-up window
41. Can you attach an lov to a field at design time?
Yes.
42. Can you attach an lov to a field at run-time? if yes, give the build-in name.
Yes. Set_item_proprety
43. What is the built-in used for showing lov at runtime?
Show_lov
44. What is the built-in used to get and set lov properties during run-time?
Get_lov_property
Set_lov_property
Record Group
45. What is a record Group?
A record group is an internal oracle forms data structure that has a simillar column/row frame work to a
database table
46. What are the different type of a record group?
Query record group
Static record group
Non query record group
47. Give built-in routine related to a record groups?
Create_group (Function)
Create_group_from_query(Function)
Delete_group(Procedure)
41
Add_group_column(Function)
Add_group_row(Procedure)
Delete_group_row(Procedure)
Populate_group(Function)
Populate_group_with_query(Function)
Set_group_Char_cell(procedure)
48. What is the built_in routine used to count the no of rows in a group?
Get_group _row_count
System Variables
49. List system variables available in forms 4.0, and not available in forms 3.0?
System.cordination_operation
System Date_threshold
System.effective_Date
System.event_window
System.suppress_working
50. System.effective_date system variable is read only True/False
False
51. What is a library in Forms 4.0?
A library is a collection of Pl/SQL program units, including user named procedures, functions & packages
52. Is it possible to attach same library to more than one form?
Yes
53. Explain the following file extention related to library?
.pll,.lib,.pld
The library pll files is a portable design file comparable to an fmb form file
The library lib file is a plat form specific, generated library file comparable to a fmx form file
The pld file is Txt format file and can be used for source controlling your library files
Parameter
54. How do you pass the parameters from one form to another form?
To pass one or more parameters to a called form, the calling form must perform the following steps in a
trigger or user named routine excute the create_parameter_list built_in function to programatically.
Create a parameter list to execute the add parameter built_in procedure to add one or more parameters list.
Execute the call_form, New_form or run_product built_in procedure and include the name or id of the
parameter list to be passed to the called form.
54. What are the built-in routines is available in forms 4.0 to create and manipulate a parameter list?
Add_parameter
Create_Parameter_list
Delete_parameter
Destroy_parameter_list
Get_parameter_attr
Get_parameter_list
set_parameter_attr
42
55. What are the two ways to incorporate images into a oracle forms application?
Boilerplate Images
Image_items
56. How image_items can be populate to field in forms 4.0?
A fetch from a long raw database column PL/Sql assignment to executing the read_image_file built_in procedure
to get an image from the file system.
57. What are the triggers associated with the image item?
When-Image-activated(Fires when the operator double clicks on an image Items)
When-image-pressed(fires when the operator selects or deselects the image item)
58. List some built-in routines used to manipulate images in image_item?
Image_add
Image_and
Image_subtract
Image_xor
Image_zoom
59. What are the built_in used to trapping errors in forms 4?
Error_type return character
Error_code return number
Error_text return char
Dbms_error_code return no.
Dbms_error_text return char
60. What is a predefined exception available in forms 4.0?
Raise form_trigger_failure
61. What are the menu items that oracle forms 4.0 supports?
Plain, Check,Radio, Separator, Magic


FORMS4.5
object groups
01. what ia an object groups?
An object group is a container for a group of objects, you define an object group when you want to package
related objects. so that you copy or reference them in another modules.
02. what are the different objects that you cannot copy or reference in object groups?
objects of differnt modules
another object groups
individual block dependent items
program units.
canvas views
03. what are different types of canvas views?
content canvas views
stacked canvas views
horizontal toolbar
vertical toolbar.
43
04. explain about content canvas views?
Most Canvas views are content canvas views a content canvas view is the "base" view that occupies the entire
content pane of the window in which it is displayed.
05. Explain about stacked canvas views?
Stacked canvas view is displayed in a window on top of, or "stacked" on the content canvas view assigned to
that same window. Stacked canvas views obscure some part of the underlying content canvas view, and or
often shown and hidden programmatically.
06. Explain about horizontal, Vertical tool bar canvas views?
Tool bar canvas views are used to create tool bars for individual windows Horizontal tool bars are display at
the top of a window, just under its menu bar.
Vertical Tool bars are displayed along the left side of a window
07. Name of the functions used to get/set canvas properties?
Get_view_property, Set_view_property
Windows
07. What is relation between the window and canvas views?
Canvas views are the back ground objects on which you place the interface items (Text items), check boxes,
radio groups etc.,) and boilerplate
objects (boxes, lines, images etc.,) that operators interact with us they run your form . Each canvas views
displayed in a window.
08. What are the different modals of windows?
Modalless windows
Modal windows
09. What are modalless windows?
More than one modelless window can be displayed at the same time, and operators can navigate among them
if your application allows them to do so . On most GUI platforms, modelless windows can also be layered to
appear either in front of or behind other windows.
10. What are modal windows?
Modal windows are usually used as dialogs, and have restricted functionality compared to modelless
windows. On some platforms for example operators cannot resize, scroll or iconify a modal window.
11. How do you display console on a window?
The console includes the status line and message line, and is displayed at the bottom of the window to which it is
assigned.
To specify that the console should be displayed, set the console window form property to the name of any
window in the form. To include the console, set console window to Null.
12. What is the remove on exit property?
For a modelless window, it determines whether oracle forms hides the window automatically when the
operators navigates to an item in the another window.
13. How many windows in a form can have console?
Only one window in a form can display the console, and you cannot chage the console assignment at runtime.
14. Can you have more than one content canvas view attached with a window?
44
Yes.
Each window you create must have atleast one content canvas view assigned to it. You can also create a
window that has manipulate contant canvas view. At run time only one of the content canvas views assign to a
window is displayed at a time.
15. What are the different window events activated at runtimes?
When_window_activated
When_window_closed
When_window_deactivated
When_window_resized
Within this triggers, you can examine the built in system variable system.event_window to determine the
name of the window for which the trigger fired.
Modules
27. What are different types of modules available in oracle form?
Form module - a collection of objects and code routines
Menu modules - a collection of menus and menu item commands that together make up an application menu
library module - a collectio of user named procedures, functions and packages that can be called from other
modules in the application
18. What are the default extensions of the files careated by forms modules?
.fmb - form module binary
.fmx - form module executable
19. What are the default extentions of the files created by menu module?
.mmb, .mmx
20 What are the default extension of the files created by library module?
The default file extensions indicate the library module type and storage format
.pll - pl/sql library module binary
Master Detail
21. What is a master detail relationship?
A master detail relationship is an association between two base table blocks- a master block and a detail
block. The relationship between the blocks reflects a primary key to foreign key relationship between the
tables on which the blocks are based.
22. What is coordination Event?
Any event that makes a different record in the master block the current record is a coordination causing event.
23. What are the two phases of block coordination?
There are two phases of block coordination: the clear phase and the population phase. During, the clear
phase, Oracle Forms navigates internally to the detail block and flushes the obsolete detail records. During
the population phase, Oracle Forms issues a SELECT statement to repopulate the detail block with detail
records associated witjh the new master record. These operations are accomplished through the execution of
triggers.
24. What are Most Common types of Complex master-detail relationships?
There are three most common types of complex master-detail relationships:
master with dependent details
master with independent details
45
detail with two masters
25. What are the different types of Delete details we can establish in Master-Details?
Cascade
Isolate
Non-isolote
26. What are the different defaust triggers created when Master Deletes Property is set to Non-isolated?
Master Delets Property Resulting Triggers
----------------------------------------------------
Non-Isolated(the default) On-Check-Delete-Master
On-Clear-Details
On-Populate-Details
26. Whar are the different default triggers created when Master Deletes Property is set to Cascade?
Ans: Master Deletes Property Resulting Triggers
---------------------------------------------------
Cascading On-Clear-Details
On-Populate-Details
Pre-delete
28. What are the different default triggers created when Master Deletes Property is set to isolated?
Master Deletes Property Resulting Triggers
---------------------------------------------------
Isolated On-Clear-Details
On-Populate-Details
29. What are the Coordination Properties in a Master-Detail relationship?
The coordination properties are
Deferred
Auto-Query
These Properties determine when the population phase of block
coordination should occur.
30. What are the different types of Coordinations of the Master with the Detail block?
42. What is the User-Named Editor?
A user named editor has the same text editing functionality as the default editor, but, becaue it is a named object,
you can specify editor attributes such as windows display size, position, and title.
43. What are the Built-ins to display the user-named editor?
A user named editor can be displayed programmatically with the built in procedure SHOW-EDITOR,
EDIT_TETITEM independent of any particular text item.
44. What is the difference between SHOW_EDITOR and EDIT_TEXTITEM?
Show editor is the generic built_in which accepts any editor name and takes some input string and returns
modified output string. Whereas the edit_textitem built_in needs the input focus to be in the text item before
the built_in is excuted.
45. What is an LOV?
46
An LOV is a scrollable popup window that provides the operator with either a single or multi column selection
list.
46. What is the basic data structure that is required for creating an LOV?
Record Group.
47. What is the "LOV of Validation" Property of an item? What is the use of it?
When LOV for Validation is set to True, Oracle Forms compares the current value of the text item to the values
in the first column displayed in the LOV.
Whenever the validation event occurs.
If the value in the text item matches one of the values in the first column of the LOV, validation succeeds,
the LOV is not displayed, and processing continues normally.
If the value in the text item does not match one of the values in the first column of the LOV, Oracle Forms
displays the LOV and uses the text item value as the search criteria to automatically reduce the list.
48. What are the built_ins used the display the LOV?
Show_lov
List_values
49. What are the built-ins that are used to Attach an LOV programmatically to an item?
set_item_property
get_item_property
(by setting the LOV_NAME property)
50. What are the built-ins that are used for setting the LOV properties at runtime?
get_lov_property
set_lov_property
51. What is a record group?
A record group is an internal Oracle Forms that structure that hs a column/row framework similar to a
database table. However, unlike database tables, record groups are separate objects that belong to the form
module which they are defined.
52. How many number of columns a record group can have?
A record group can have an unlimited number of columns of type CHAR, LONG, NUMBER, or DATE
provided that the total number of column does not exceed 64K.
53. What is the Maximum allowed length of Record group Column?
Record group column names cannot exceed 30 characters.
54. What are the different types of Record Groups?
Query Record Groups
NonQuery Record Groups
State Record Groups
55. What is a Query Record Group?
A query record group is a record group that has an associated SELECT statement. The columns in a query
record group derive their default names, data types, had lengths from the database columns referenced in the
SELECT statement. The records in query record group are the rows retrieved by the query associated with that
record group.
56. What is a Non Query Record Group?
47
A non-query record group is a group that does not have an associated query, but whose structure and values can
be modified programmatically at runtime.
57. What is a Static Record Group?
A static record group is not associated with a query, rather, you define its structure and row values at design
time, and they remain fixed at runtime.
58. What are the built-ins used for Creating and deleting groups?
CREATE-GROUP (function)
CREATE_GROUP_FROM_QUERY(function)
DELETE_GROUP(procedure)
59.What are the built -ins used for Modifying a group's structure?
ADD-GROUP_COLUMN (function)
ADD_GROUP_ROW (procedure)
DELETE_GROUP_ROW(procedure)
60. POPULATE_GROUP(function)
POPULATE_GROUP_WITH_QUERY(function)
SET_GROUP_CHAR_CELL(procedure)
SET_GROUP_DATE_CELL(procedure)
SET_GROUP_NUMBER_CELL(procedure)
61. What are the built-ins used for Getting cell values?
GET_GROUP_CHAR_CELL (function)
GET_GROUP_DATE_CELL(function)
GET_GROUP_NUMBET_CELL(function)
62. What are built-ins used for Processing rows?
GET_GROUP_ROW_COUNT(function)
GET_GROUP_SELECTION_COUNT(function)
GET_GROUP_SELECTION(function)
RESET_GROUP_SELECTION(procedure)
SET_GROUP_SELECTION(procedure)
UNSET_GROUP_SELECTION(procedure)
63. What are the built-ins used for finding Object ID function?
FIND_GROUP(function)
FIND_COLUMN(function)
64. Use the ADD_GROUP_COLUMN function to add a column to a record group that was created at design
time.
I) TRUE II)FALSE
II) FALSE
65. Use the ADD_GROUP_ROW procedure to add a row to a static record group
I) TRUE II)FALSE
I) FALSE
61. What are the built-in used for getting cell values?
Get_group_char_cell(function)
Get_group_date_cell(function)
Get_group_number_cell(function)
48
62. What are the built-ins used for processing rows?
Get_group_row_count(function)
Get_group_selection_count(function)
Get_group_selection(function)
Reset_group_selection(procedure)
Set_group_selection(procedure)
Unset_group_selection(procedure)
63. What are the built-ins used for finding object ID functions?
Find_group(function)
Find_column(function)
64. Use the add_group_column function to add a column to record group that was created at a design time?
False.
65. Use the Add_group_row procedure to add a row to a static record group 1. true or false?
False.
parameters
66. What are parameters?
Parameters provide a simple mechanism for defining and setting the values
of inputs that are required by a form at startup. Form parameters are variables of type char,number,date that
you define at design time.
67. What are the Built-ins used for sending Parameters to forms?
You can pass parameter values to a form when an application executes the call_form, New_form, Open_form or
Run_product.
68. What is the maximum no of chars the parameter can store?
The maximum no of chars the parameter can store is only valid for char parameters, which can be upto 64K.
No parameters default to 23Bytes and Date parameter default to 7Bytes.
69. How do you call other Oracle Products from Oracle Forms?
Run_product is a built-in, Used to invoke one of the supported oracle tools products and specifies the name of the
document or module to be run. If the called product is unavailable at the time of the call, Oracle Forms returns a
message to the opertor.
70. How do you reference a Parameter?
In Pl/Sql, You can reference and set the values of form parameters using bind variables syntax. Ex.
PARAMETER name = '' or :block.item = PARAMETER
Parameter name
71. How do you reference a parameter indirectly?
To indirectly reference a parameter use the NAME IN, COPY 'built-ins to indirectly set and reference the
parameters value' Example name_in ('capital parameter my param'), Copy ('SURESH','Parameter my_param')
72. What are the different Parameter types?
Text Parameters
Data Parameters
73. When do you use data parameter type?
49
When the value of a data parameter being passed to a called product is always the name of the record group
defined in the current form. Data parameters are used to pass data to produts invoked with the run_product
built-in subprogram.
74. Can you pass data parametrs to forms?
No.
IMAGES
75. What are different types of images?
Boiler plate images
Image Items
76. What is the difference between boiler plat images and image items?
Boiler plate Images are static images (Either vector or bit map) that you import from the file system or database
to use a grapical elements in your form, such as company logos and maps Image items are special types of
interface controls that store and display either vector or bitmap images. Llike other items that store values,
image items can be either base table items(items that relate directly to database columns) or control items. The
definition of an image item is stored as part of the form module FMB and FMX files, but no image file is
actualy associated with an image item until the item is populate at run time.
77. What are the trigger associated with image items?
When-image-activated fires when the operators double clicks on an image item when-image-pressed
fires when an operator clicks or double clicks on an image item
78. What is the use of image_zoom built-in?
To manipulate images in image items.
WORKING WITH MULTIPLE FORMS
79. How do you create a new session while open a new form?
Using open_form built-in setting the session option Ex. Open_form('Stocks ',active,session). when invoke the
mulitiple forms with open form and call_form in the same application, state whether the following are
true/False
80. Any attempt to navigate programatically to disabled form in a call_form stack is allowed?
False
81. An open form can not be execute the call_form procedure if you chain of called forms has been initiated by
another open form?
True
82. When a form is invoked with call_form, Does oracle forms issues a save point?
True
Mouse Operations
83. What are the various sub events a mouse double click event involves?
Double clicking the mouse consists of the mouse down, mouse up, mouse click, mouse down & mouse up
events.
84, State any three mouse events system variables?
System.mouse_button_pressed
50
System.mouse_button_shift_state
system.mouse_item
system.mouse_canvas
system.mouse_record
OLE
85. What is an OLE?
Object Linking & Embadding provides you with the capability to integrate objects from many Ms-Windows
applications into a single compound document creating integrated applications enables you to use the features
form .
86. What is the difference between object embedding & linking in Oracle forms?
In Oracle forms, Embedded objects become part of the form module, and linked objects are references from a
form module to a linked source file.
87. What is the difference between OLE Server & Ole Container?
An Ole server application creates ole Objects that are embedded or linked in ole Containers ex. Ole servers are
ms_word & ms_excell. OLE containers provide a place to store, display and manipulate objects that are created
by ole server applications. Ex. oracle forms is an example of an ole Container.
88. What are the different styles of actvation of ole Objects?
In place activation
External activation
ViSUAL Attributes & property clauses
89. What are visual attributes?
Visual attributes are the font, color, pattern proprities that you set for form and menu objects that appear in your
application interface.
90. What is a property clause?
A property clause is a named object that contains a list of properties and thier settings. Once you create a property
clause you can base other object on it. An object based on a property can inherit the setting of any property
in the clause that makes sense for that object.
91. Can a property clause itself be based on a property clause?
Yes
92. What are the important difference between property clause and visual attributes?
Named visual attributes differed only font, color & pattern attributes, property clauses can contain this and any
other properties. You can change the appearance of objects at run time by changing the named visual
attributes programatically , property clause assignments cannot be changed programatically. When an object is
inheriting from both a property clause and named visual attribute, the named visual attribute settings take
precedence, and any visual attribute properties in the class are ignored.
Form Build-ins
93. What is a Text_io Package?
It allows you to read and write information to a file in the file system.
94. What is an User_exit?
51
Calls the user exit named in the user_exit_string. Invokes a 3Gl programe by name which has been properly
linked into your current oracle forms executable.
95. What is synchronize?
It is a terminal screen with the internal state of the form. It updates the screen display to reflect the information
that oracle forms has in its internal representation of the screen.
96. What is forms_DDL?
Issues dynamic Sql statements at run time, including server side pl/SQl and DDL
Triggers
97. What is WHEN-Database-record trigger?
Fires when oracle forms first marks a record as an insert or an update. The trigger fires as soon as oracle forms
determines through validation that the record should be processed by the next post or commit as an insert or
update. c generally occurs only when the operators modifies the first item in the record, and after the operator
attempts to navigate out of the item.
98. What are the master-detail triggers?
On-Check_delete_master
On_clear_details
On_populate_details
99. What is the difference between $$DATE$$ & $$DBDATE$$
$$DBDATE$$ retrieves the current database date
$$date$$ retrieves the current operating system date.
100. What is system.coordination_operation?
It represents the coordination causing event that occur on the master block in master-detail relation.
101. What are the difference between lov & list item?
Lov is a property where as list item ias an item. A list item can have only one column, lov can have one or more
columns.
102. What are the different display styles of list items?
Pop_list
Text_list
Combo box
103. What is pop list?
The pop list style list item appears initially as a single field (similar to a text item field). When the operator
selects the list icon, a list of available choices appears.
104. What is a text list?
The text list style list item appears as a rectangular box which displays the fixed number of values. When the text
list contains values that can not be displayed, a vertical scroll bar appears, allowing the operator to view and
select undisplayed values.
105. What is a combo box?
A combo box style list item combines the features found in list and text item. Unlike the pop list or the text list
style list items, the combo box style list item will both display fixed values and accept one operator entered
value.
52
106. What are display items?
Display items are similar to text items with the exception that display items only store and display fetched or
assigned values.Display items are generaly used as boilerplate or conditional text.
107. What is difference between open_form and call_form?
when one form invokes another form by executing open_form the first form remains displayed,and operators
can navigate between the forms as desired. when one form invokes another form by executing call_form,the
called form is modal with respect to the calling form.That is, any windows that belong to the calling form are
disabled, and operators cannot navigate to them until they first exit the called form.
108. What is new_form built-in?
When one form invokes another form by executing new_form oracle form exits the first form and releases its
memory before loading the new form calling new form completely replace the first with the second. If there are
changes pending in the first form,the operator will be prompted to save them before the new form is loaded.
109. What is a library?
A library is a collection of subprograms including user named procedures, functions and packages.
110. What is the advantage of the library?
Library's provide a convenient means of storing client-side program units and sharing them among multipule
applications. Once you create a library, you can attach it to any other form,menu,or library modules. When you
can call library program units from triggers menu items commands and user named routine, you write in the
modules to which you have attach the library.
when a library attaches another library ,program units in the first library can reference program units in the
attached library. Library support dynamic loading-that is library program units are loaded into an
application only when needed. This can significantly reduce the run-time memory requirements of an
applications.
111. What is strip sources generate options?
Removes the source code from the library file and generates a library files that contains only pcode.The
resulting file can be used for final deployment, but can not be subsequently edited in the designer.
ex. f45gen module=old_lib.pll userid=scott/tiger
strip_source YES output_file
112.What are the vbx controls?
Vbx control provide a simple mehtod of buildig and enhancing user interfaces.The controls can use to
obtain user inputs and display program outputs.vbx control where originally develop as extensions for the ms
visual basic environments and include such items as sliders,grides and knobs.
113. What is a timer?
Timer is a "internal time clock" that you can programmatically create to perform an action each time the timer
expires.
114. What are built-ins associated with timers?
find_timer
create_timer
delete_timer
115. what are difference between post database commit and post-form commit?
Post-form commit fires once during the post and commit transactions process, after the database commit
occures. The post-form-commit trigger fires after inserts,updates and deletes have been posted to the database
but before the transactions have been finalished in the issuing the command.The post-database-commit
trigger fires after oracle forms issues the commit to finalished transactions.
53
116. What is a difference between pre-select and pre-query?
Fires during the execute query and count query processing after oracle forms constructs the select statement
to be issued, but before the statement is actually issued.
The pre-query trigger fires just before oracle forms issues the select statement to the database after the
operator as define the example records by entering the query criteria in enter query mode.
Pre-query trigger fires before pre-select trigger.
117. What is trigger associated with the timer?
When-timer-expired.
118 What is the use of transactional triggers?
Using transactional triggers we can control or modify the default functionality of the oracle forms.

REPORTS
1. What are the different file extensions that are created by oracle reports?
Rep file and Rdf file.
2. From which designation is it preferred to send the output to the printed?
Previewer.
3. Is it possible to disable the parameter from while running the report?
Yes
4. What is lexical reference?How can it be created?
Lexical reference is place_holder for text that can be embedded in a sql
statements.A lexical reference can be created using & before the column or
parameter name.
5. What is bind reference and how can it carate?
Bind reference are used to replace the single value in sql,pl/sql
statements a bind reference can be careated using a (:) before a column or
a parameter name.
6.What use of command line parameter cmd file?
It is a command line argument that allows you to specify a file that contain a set of arguments for r20run.
7.Where is a procedure return in an external pl/sql library executed at the client or at the server?
At the client.
8. Where is the external query executed at the client or the server?
At the server.
9. What are the default parameter that appear at run time in the parameter screen?
Destype and Desname.
10. Which parameter can be used to set read level consistency across multiple queries?
Read only.
11. What is term?
The term is terminal definition file that describes the terminal form which you are using r20run.
54
12. What is use of term?
The term file which key is correspond to which oracle report functions.
13. Is it possible to insert comments into sql statements return in the data model editor?
Yes.
14. If the maximum record retrieved property of the query is set to 10 then a summary value will be calculated?
Only for 10 records.
15. What are the sql clauses supported in the link property sheet?
Where startwith having.
16. To execute row from being displayed that still use column in the row which property can be used?
Format trigger.
17. Is it possible to set a filter condition in a cross product group in matrix reports?
No.
18.if a break order is set on a column would it effect columns which are under the column?
No.
19. With which function of summary item is the compute at options required?
percentage of total functions.
20. What is the purpose of the product order option in the column property sheet?
To specify the order of individual group evaluation in a cross products.
21.Can a formula column be obtained through a select statement?
Yes.
22.Can a formula column refered to columns in higher group?
Yes.
23. How can a break order be created on a column in an existing group?
By dragging the column outside the group.
24. What are the types of calculated columns available?
Summary, Formula, Placeholder column.
25. What is the use of place holder column?
A placeholder column is used to hold a calculated values at a specified place rather than allowing is to appear
in the actual row where it has to appeared.
26. What is the use of hidden column?
A hidden column is used to when a column has to embedded into boilerplate text.
27. What is the use of break group?
A break group is used to display one record for one group ones.While multiple related records in other group
can be displayed.
28. If two groups are not linked in the data model editor, What is the hierarchy between them?
Two group that is above are the left most rank higher than the group that is to right or below it.
29.The join defined by the default data link is an outer join yes or no?
55
Yes.
30. How can a text file be attached to a report while creating in the report writer?
By using the link file property in the layout boiler plate property sheet.
31. Can a repeating frame be careated without a data group as a base?
No.
32. Can a field be used in a report wihtout it appearing in any data group?
Yes.
33. For a field in a repeating frame, can the source come from the column which does not exist in the data group
which forms the base for the frame?
Yes.
34. Is it possible to center an object horizontally in a repeating frame that has a variable horizontal size?
Yes.
35. If yes,how?
By the use anchors.
36. What are the two repeating frame always associated with matrix object?
One down repeating frame below one across repeating frame.
37. Is it possible to split the printpreviewer into more than one region?
Yes.
38. Does a grouping done for objects in the layout editor affect the grouping done in the datamodel editor?
No.
39. How can a square be drawn in the layout editor of the report writer?
By using the rectangle tool while pressing the (Constraint) key.
40. To display the page no. for each page on a report what would be the source & logical page no. or & of
physical page no.?
& physical page no.
41. What does the term panel refer to with regard to pages?
A panel is the no. of physical pages needed to print one logical page.
42. What is an anchoring object & what is its use?
An anchoring object is a print condition object which used to explicitly or implicitly anchor other objects to itself.
43. What is a physical page? & What is a logical page?
A physical page is a size of a page. That is output by the printer. The
logical page is the size of one page of the actual report as seen in the
Previewer.
44. What is the frame & repeating frame?
A frame is a holder for a group of fields. A repeating frame is used to
display a set of records when the no. of records that are to displayed is
not known before.
56
REPORT TRIGGERS.
45. What are the triggers available in the reports?
Before report, Before form, After form , Between page, After report.
46. Does a Before form trigger fire when the parameter form is suppressed.
Yes.
47. At what point of report execution is the before Report trigger fired?
After the query is executed but before the report is executed and the
records are displayed.
48. Is the After report trigger fired if the report execution fails?
Yes.
49. Give the sequence of execution of the various report triggers?
Before form , After form , Before report, Between page, After report.
50. Is it possible to modify an external query in a report which contains
it?
No.
51. What are the ways to monitor the performance of the report?
Use reports profile executable statement.
Use SQL trace facility.
52. Why is it preferable to create a fewer no. of queries in the data
model.
Because for each query, report has to open a separate cursor and has to
rebind, execute and fetch data.
53. What are the various methods of performing a calculation in a report?
1. Perform the calculation in the SQL statements itself.
2. Use a calculated / summary column in the data model.
54. Which of the above methods is the faster method?
performing the calculation in the query is faster.
55. Why is a Where clause faster than a group filter or a format trigger?
Because, in a where clause the condition is applied during data retrieval
than after retrieving the data.
56. What is the main diff. bet. Reports 2.0 & Reports 2.5?
Report 2.5 is object oriented.
57. What is the diff. bet. setting up of parameters in reports 2.0 reports
2.5?
LOVs can be attached to parameters in the reports 2.5 parameter form.
58. How is link tool operation different bet. reports 2 & 2.5?
In Reports 2.0 the link tool has to be selected and then two fields to be
linked are selected and the link is automatically created. In 2.5 the first
field is selected and the link tool is then used to link the first field to
57
the second field.
REPORT 2.5 SPECIFIC ISSUES.
59.What are the two types views available in the object navigator(specific
to report 2.5)?
View by structure and view by type .
60. Which of the two views should objects according to possession?
view by structure.
61.How is possible to restrict the user to a list of values while entering
values for parameters?
By setting the Restrict To List property to true in the parameter property
sheet.
62. How is it possible to select generate a select ste. for the query in
the query property sheet?
By using the tables/columns button and then specifying the table and the
column names.
63. If a parameter is used in a query without being previously defined,
what diff. exist betw. report 2.0 and 2.5 when the query is applied?
While both reports 2.0 and 2.5 create the parameter, report 2.5 gives a
message that a bind parameter has been created.
64. Do user parameters appear in the data modal editor in 2.5?
No.
65.What is the diff. when confine mode is on and when it is off?
When confine mode is on, an object cannot be moved outside its parent in
the layout.
66. What is the diff. when Flex mode is mode on and when it is off?
When flex mode is on, reports automatically resizes the parent when the
child is resized.
67. How can a button be used in a report to give a drill down facility?
By setting the action asscoiated with button to Execute pl/sql option and
using the SRW.Run_report function.
68. What are the two ways by which data can be generated for a parameter's
list of values?
1. Using static values.
2. Writing select statement.
69. What are the two panes that Appear in the design time pl/sql
interpreter?
1.Source pane. 2. Interpreter pane
70. What are three panes that appear in the run time pl/sql interpreter?
1.Source pane. 2. interpreter pane. 3. Navigator pane.
58
CROSS PRODUCTS AND MATRIX REPORTS
71. How can a cross product be created?
By selecting the cross products tool and drawing a new group surrounding
the base group of the cross products.
72. How can a group in a cross products be visually distinguished from a
group that does not form a cross product?
A group that forms part of a cross product will have a thicker border.
73. Atleast how many set of data must a data model have before a data model
can be base on it?
Four.
74. Is it possible to have a link from a group that is inside a cross
product to a group outside? (Y/N)
No.
75. Is it possible to link two groups inside a cross products after the
cross products group has been created?
No.
76. What is an user exit used for?
A way in which to pass control (and possibly arguments ) form Oracle report
to another Oracle products of 3 GL and then return control ( and ) back
to Oracle reprots.
77. What are the three types of user exits available?
Oracle Precompiler exits, Oracle call interface,NonOracle user exits.
78. How can values be passed bet. precompiler exits & Oracle call
interface?
By using the statement EXECIAFGET & EXECIAFPUT.
79. How can I message to passed to the user from reports?
By using SRW.MESSAGE function.

MCQ


1. SNAPSHOT is used for
[DBA] a] Synonym, b] Table space, c] System server, d] Dynamic data replication
Ans : D
2. We can create SNAPSHOTLOG for
[DBA] a] Simple snapshots, b] Complex snapshots, c] Both A & B, d] Neither A nor B
Ans : A
3. Transactions per rollback segment is derived from
[DBA] a] Db_Block_Buffers, b] Processes, c] Shared_Pool_Size, d] None
of the above
Ans : B
4. ENQUEUE resources parameter information is derived from
[DBA] a] Processes or DDL_LOCKS and DML_LOCKS, b] LOG_BUFFER,
c] DB__BLOCK_SIZE..
Ans : A
5. LGWR process writes information into
a] Database files, b] Control files, c] Redolog files, d] All the
above.
Ans : C
6. SET TRANSACTION USE ROLLBACK SEGMENT is used to create user objects in a particular Tablespace
a] True, b] False
Ans : False
7. Databases overall structure is maintained in a file called
a] Redolog file, b] Data file, c] Control file, d] All of the above.
Ans : C
8. These following parameters are optional in init.ora parameter file DB_BLOCK_SIZE,
PROCESSES
a] True, b] False
Ans : False
9. Constraints cannot be exported through EXPORT command
a] True, b] False
Ans : False
10. It is very difficult to grant and manage common privileges needed by different groups of
database users using the roles
a] True, b] False
Ans : False
11. What is difference between a DIALOG WINDOW and a DOCUMENT WINDOW
regarding moving the window with respect to the application window
a] Both windows behave the same way as far as moving the window is concerned.
b] A document window can be moved outside the application window while a dialog window cannot be moved
c] A dialog window can be moved outside the application window while a document window cannot be moved
Ans : C
12. What is the difference between a MESSAGEBOX and an ALERT
a] A messagebox can be used only by the system and cannot be used in user application while an alert can be used in user application also.
b] A alert can be used only by the system and cannot be use din user application
while an messagebox can be used in user application also.
c] An alert requires an response from the userwhile a messagebox just flashes a message
and only requires an acknowledment from the user
d] An message box requires an response from the userwhile a alert just flashes a message an only requires an acknowledment from the user
Ans : C
13. Which of the following is not an reason for the fact that most of the processing is done at the server?
a] To reduce network traffic. b] For application sharing, c] To
implement business rules
centrally, d] None of the above
Ans : D
14. Can a DIALOG WINDOW have scroll bar attached to it?
a] Yes, b] No
Ans : B
15. Which of the following is not an advantage of GUI systems?
a] Intuitive and easy to use., b] GUI's can display multiple
applications in multiple windows
c] GUI's provide more user interface objects for a developer
d] None of the above
Ans :D
16. What is the difference between a LIST BOX and a COMBO BOX?
a] In the list box, the user is restricted to selecting a value from a list but in a combo box the user can type in a value which is not in the list
b] A list box is a data entry area while a combo box can be used only for control purposes
c] In a combo box, the user is restricted to selecting a value from a list but in a list box the user can type in a value which is not in the list
d] None of the above
Ans : A
17. In a CLIENT/SERVER environment , which of the following would not be
done at the client?
a] User interface part, b] Data validation at entry line, c]
Responding to user events,
d] None of the above
Ans : D
18. Why is it better to use an INTEGRITY CONSTRAINT to validate data in a
table than to use a STORED PROCEDURE?
a] Because an integrity constraint is automatically checked while data is inserted into or updated in a table while a stored procedure has to be specifically invoked
b] Because the stored procedure occupies more space in the database than a integrity constraint definition
c] Because a stored procedure creates more network traffic than a integrity constraint definition
Ans : A
19. Which of the following is not an advantage of a client/server model?
a] A client/server model allows centralised control of data and centralised implementation of business rules.
b] A client/server model increases developer;s productivity
c] A client/server model is suitable for all applications
d] None of the above.
Ans : C
20. What does DLL stands for?
a] Dynamic Language Library
b] Dynamic Link Library
c] Dynamic Load Library
d] None of the above
Ans : B
21. POST-BLOCK trigger is a
a] Navigational trigger
b] Key trigger
c] Transactional trigger
d] None of the above
Ans : A
22. The system variable that records the select statement that SQL * FORMS
most recently used to populate a block is
a] SYSTEM.LAST_RECORD
b] SYSTEM.CURSOR_RECORD
c] SYSTEM.CURSOR_FIELD
d] SYSTEM.LAST_QUERY
Ans: D
23. Which of the following is TRUE for the ENFORCE KEY field
a] ENFORCE KEY field characterstic indicates the source of the value
that SQL*FORMS
uses to populate the field
b] A field with the ENFORCE KEY characterstic should have the INPUT
ALLOWED
charaterstic turned off
a] Only 1 is TRUE
b] Only 2 is TRUE
c] Both 1 and 2 are TRUE
d] Both 1 and 2 are FALSE
Ans : A
24. What is the maximum size of the page?
a] Characters wide & 265 characters length
b] Characters wide & 265 characters length
c] Characters wide & 80 characters length
d] None of the above
Ans : B
25. A FORM is madeup of which of the following objects
a] block, fields only,
b] blocks, fields, pages only,
c] blocks, fields, pages, triggers and form level procedures,
d] Only blocks.
Ans : C
26. For the following statements which is true
1] Page is an object owned by a form
2] Pages are a collection of display information such as constant text and graphics.
a] Only 1 is TRUE
b] Only 2 is TRUE
c] Both 1 & 2 are TRUE
d] Both are FALSE
Ans : B
27. The packaged procedure that makes data in form permanent in the Database is
a] Post
b] Post form
c] Commit form
d] None of the above
Ans : C
28. Which of the following is TRUE for the SYSTEM VARIABLE $$date$$
a] Can be assigned to a global variable
b] Can be assigned to any field only during design time
c] Can be assigned to any variable or field during run time
d] None of the above
Ans : B
29. Which of the following packaged procedure is UNRESTRICTED?
a] CALL_INPUT, b] CLEAR_BLOCK, c] EXECUTE_QUERY, d] USER_EXIT
Ans : D
30. Identify the RESTRICTED packaged procedure from the following
a] USER_EXIT, b] MESSAGE, c] BREAK, d] EXIT_FORM
Ans : D
31. What is SQL*FORMS
a] SQL*FORMS is a 4GL tool for developing & executing Oracle based interactive applications.
b] SQL*FORMS is a 3GL tool for connecting to the Database.
c] SQL*FORMS is a reporting tool
d] None of the above.
Ans : A
32. Name the two files that are created when you generate a form using
Forms 3.0
a] FMB & FMX,
b] FMR & FDX,
c] INP & FRM,
d] None of the above
Ans : C
33. What is a trigger
a] A piece of logic written in PL/SQL
b] Executed at the arrival of a SQL*FORMS event
c] Both A & B
d] None of the above
Ans : C
34. Which of the folowing is TRUE for a ERASE packaged procedure
1] ERASE removes an indicated Global variable & releases the memory
associated with it
2] ERASE is used to remove a field from a page
1] Only 1 is TRUE
2] Only 2 is TRUE
3] Both 1 & 2 are TRUE
4] Both 1 & 2 are FALSE
Ans : 1
35. All datafiles related to a Tablespace are removed when the Tablespace is dropped
a] TRUE
b] FALSE
Ans : B
36. Size of Tablespace can be increased by
a] Increasing the size of one of the Datafiles
b] Adding one or more Datafiles
c] Cannot be increased
d] None of the above
Ans : B
37. Multiple Tablespaces can share a single datafile
a] TRUE
b] FALSE
Ans : B
38. A set of Dictionary tables are created
a] Once for the Entire Database
b] Every time a user is created
c] Every time a Tablespace is created
d] None of the above
Ans : A
64
39. Datadictionary can span across multiple Tablespaces
a] TRUE
b] FALSE
Ans : B
40. What is a DATABLOCK
a] Set of Extents
b] Set of Segments
c] Smallest Database storage unit
d] None of the above
Ans : C
41. Can an Integrity Constraint be enforced on a table if some existing
table data does not satisfy
the constraint
a] Yes
b] No
Ans : B
42. A column defined as PRIMARY KEY can have NULL's
a] TRUE
b] FALSE
Ans : B
43. A Transaction ends
a] Only when it is Committed
b] Only when it is Rolledback
c] When it is Committed or Rolledback
d] None of the above
Ans : C
44. A Database Procedure is stored in the Database
a] In compiled form
b] As source code
c] Both A & B
d] Not stored
Ans : C
45. A database trigger doesnot apply to data loaded before the definition
of the trigger
a] TRUE
b] FALSE
Ans : A
46. Dedicated server configuration is
a] One server process - Many user processes
b] Many server processes - One user process
c] One server process - One user process
d] Many server processes - Many user processes
Ans : C

47. Which of the following does not affect the size of the SGA
a] Database buffer
b] Redolog buffer
c] Stored procedure
d] Shared pool
Ans : C
48. What does a COMMIT statement do to a CURSOR
a] Open the Cursor
b] Fetch the Cursor
c] Close the Cursor
d] None of the above
Ans : D
49. Which of the following is TRUE
1] Host variables are declared anywhere in the program
2] Host variables are declared in the DECLARE section
a] Only 1 is TRUE
b] Only 2 is TRUE
c] Both 1 & 2are TRUE
d] Both are FALSE
Ans : B
50. Which of the following is NOT VALID is PL/SQL
a] Bool boolean;
b] NUM1, NUM2 number;
c] deptname dept.dname%type;
d] date1 date := sysdate
Ans : B
51. Declare
fvar number := null; svar number := 5
Begin
goto << fproc>>
if fvar is null then
<< fproc>>
svar := svar + 5
end if;
End;
What will be the value of svar after the execution?
a] Error
b] 10
c] 5
d] None of the above
Ans : A
52. Which of the following is not correct about an Exception?
a] Raised automatically / Explicitly in response to an ORACLE_ERROR
b] An exception will be raised when an error occurs in that block
c] Process terminates after completion of error sequence.
d] A Procedure or Sequence of statements may be processed.
Ans : C
53. Which of the following is not correct about User_Defined Exceptions?
a] Must be declared
b] Must be raised explicitly
c] Raised automatically in response to an Oracle error
d] None of the above
Ans : C
54. A Stored Procedure is a
a] Sequence of SQL or PL/SQL statements to perform specific function
b] Stored in compiled form in the database
c] Can be called from all client environmets
d] All of the above
Ans : D
55. Which of the following statement is false
a] Any procedure can raise an error and return an user message and error number
b] Error number ranging from 20000 to 20999 are reserved for user defined messages
c] Oracle checks Uniqueness of User defined errors 
d] Raise_Application_error is used for raising an user defined error.
Ans : C
56. Is it possible to open a cursor which is in a Package in another
procedure?
a] Yes
b] No
Ans : A
57. Is it possible to use Transactional control statements in Database
Triggers?
a] Yes
b] No
Ans : B
58. Is it possible to Enable or Disable a Database trigger?
a] Yes
b] No
Ans : A
59. PL/SQL supports datatype(s)
a] Scalar datatype
b] Composite datatype
c] All of the above
d] None of the above
Ans C
60. Find the ODD datatype out
a] VARCHAR2
b] RECORD
c] BOOLEAN
d] RAW
67
Ans : B
61. Which of the following is not correct about the "TABLE" datatype?
a] Can contain any no of columns
b] Simulates a One-dimensional array of unlimited size
c] Column datatype of any Scalar type
d] None of the above
Ans : A
62. Find the ODD one out of the following
a] OPEN
b] CLOSE
c] INSERT
d] FETCH
Ans C
63. Which of the following is not correct about Cursor?
a] Cursor is a named Private SQL area
b] Cursor holds temporary results
c] Cursor is used for retrieving multiple rows
d] SQL uses implicit Cursors to retrieve rows
Ans : B
64. Which of the following is NOT VALID in PL/SQL?
a] Select ... into
b] Update
c] Create
d] Delete
Ans : C
65. What is the Result of the following 'VIK'||NULL||'RAM'?
a] Error
b] VIK RAM
c] VIKRAM
d] NULL
Ans : C
66. Declare
a number := 5; b number := null; c number := 10;
Begin
if a > b AND a < c then
a := c * a;
end if;
End;
What will be the value of 'a' after execution?
a] 50
b] NULL
c] 5
d] None of the above
Ans : C
67. Does the Database trigger will fire when the table is TRUNCATED?
a] Yes
68
b] No
Ans : B
68. SUBSTR(SQUARE ANS ALWAYS WORK HARD,14,6) will return
a] ALWAY
b} S ALWA
c] ALWAYS
Ans : C
69. REPLACE('JACK AND JUE','J','BL') will return
a] JACK AND BLUE
b] BLACK AND JACK
c] BLACK AND BLUE
d] None of the above
Ans : C
70. TRANSLATE('333SQD234','0123456789ABCDPQRST','0123456789') will return
a] 333234
b] 333333
c] 234333
d] None of the above
Ans : A
71. EMPNO ENAME SAL
A822 RAMASWAMY 3500
A812 NARAYAN 5000
A973 UMESH 2850
A500 BALAJI 5750
Use these data for the following Questions
Select SAL from EMP E1 where 3 > ( Select count(*) from Emp E2
where E1.SAL > E2.SAL ) will retrieve
a] 3500,5000,2500
b] 5000,2850
c] 2850,5750
d] 5000,5750
Ans : A
72. Is it possible to modify a Datatype of a column when column contains data?
a] Yes
b] No
Ans B
73. Which of the following is not correct about a View?
a] To protect some of the columns of a table from other users
b] Ocuupies data storage space
c] To hide complexity of a query
d] To hide complexity of a calculations
Ans : B
74. Which is not part of the Data Definiton Language?
a] CREATE
b] ALTER
c] ALTER SESSION
Ans : C
75. The Data Manipulation Language statements are
a] INSERT
b] UPDATE
c] SELECT
d] All of the above
Ans : D
76. EMPNO ENAME SAL
A822 RAMASWAMY 3500
A812 NARAYAN 5000
A973 UMESH
A500 BALAJI 5750
Using the above data
Select count(sal) from Emp will retrieve
a] 1
b] 0
c] 3
d] None of the above
Ans : C
77. If an UNIQUE KEY constraint on DATE column is created, will it accept
the rows that are
inserted with SYSDATE?
a] Will
b] Won't
Ans : B
78. What are the different events in Triggers?
a] Define, Create
b] Drop, Comment
c] Insert, Update, Delete
d] All of the above
Ans : C
79. What built-in subprogram is used to manipulate images in image items?
a] Zoom_out
b] Zoom_in'
c] Image_zoom
d] Zoom_image
Ans : C
80. Can we pass RECORD GROUP between FORMS?
a] Yes
b] No
Ans : A
81. SHOW_ALERT function returns
a] Boolean
b] Number
c] Character
d] None of the above
Ans : B
82. What SYSTEM VARIABLE is used to refer DATABASE TIME?
a] $$dbtime$$
b] $$time$$
c] $$datetime$$
d] None of the above
Ans : A
83. :SYSTEM.EFFECTIVE.DATE variable is
a] Read only
b] Read & Write
c] Write only
d] None of the above
Ans : C
84. How can you CALL Reports from Forms4.0?
a] Run_Report built_in
b] Call_Report built_in
c] Run_Product built_in
d] Call_Product built_in
Ans : C
85. When do you get a .PLL extension?
a] Save Library file
b] Generate Library file
c] Run Library file
d] None of the above
Ans : A
86. What is built_in Subprogram?
a] Stored procedure & Function
b] Collection of Subprogram
c] Collection of Packages
d] None of the above
Ans : D
87. GET_BLOCK property is a
a] Restricted procedure
b] Unrestricted procedure
c] Library function
d] None of the above
Ans : D
88. A CONTROL BLOCK can sometimes refer to a BASETABLE?
a] TRUE
b] FALSE
Ans : B
89. What do you mean by CHECK BOX?
a] Two state control
b] One state control
c] Three state control
d] none of the above
Ans : C - Please check the Correctness of this Answer ( The correct answer is 2 )
90. List of Values (LOV) supports
a] Single column
b] Multi column
c] Single or Multi column
d] None of the above
Ans : C
91. What is Library in Forms 4.0?
a] Collection of External field
b] Collection of built_in packages
c] Collection of PL/SQl functions, procedures and packages
d] Collection of PL/SQL procedures & triggers
Ans : C
92. Can we use a RESTRICTED packaged procedure in WHEN_TEXT_ITEM trigger?
a] Yes
b] No
Ans : B
93. Can we use GO_BLOCK package in a PRE_TEXT_ITEM trigger?
a] Yes
b] No
Ans : B
94. What type of file is used for porting Forms 4.5 applications to various platforms?
a] .FMB file
b] .FMX file
c] .FMT file
d] .EXE file
Ans : C
95. What built_in procedure is used to get IMAGES in Forms 4.5?
a] READ_IMAGE_FILE
b] GET_IMAGE_FILE
c] READ_FILE
d] GET_FILE
Ans A
96. When a form is invoked with CALL_FORM does Oracle forms issues
SAVEPOINT?
a] Yes
b] No
Ans : A
97. Can we attach the same LOV to different fields in Design time?
a] Yes
72
b] No
Ans : A
98. How do you pass values from one form to another form?
a] LOV
b] Parameters
c] Local variables
d] None of the above
Ans : B
99. Can you copy the PROGRAM UNIT into an Object group?
a] Yes
b] No
Ans : B
100. Can MULTIPLE DOCUMENT INTERFACE (MDI) be used in Forms 4.5?
a] Yes
b] No
Ans : A
101. When is a .FMB file extension is created in Forms 4.5?
a] Generating form
b] Executing form
c] Save form
d] Run form
Ans : C
102. What is a Built_in subprogram?
a] Library
b] Stored procedure & Function
c] Collection of Subprograms
d] None of the above
Ans : D
103. What is a RADIO GROUP?
a] Mutually exclusive
b] Select more than one column
c] Above all TRUE
d] Above all FALSE
Ans : A
104. Identify the Odd one of the following statements?
a] Poplist
b] Tlist
c] List of values
d] Combo box
Ans : C
105. What is an ALERT?
a] Modeless window
b] Modal window
c] Both are TRUE
d] None of the above
Ans : B
106. Can an Alert message be changed at runtime?
a] Yes
b] No
Ans : A
107. Can we create an LOV without an RECORD GROUP?
a} Yes
b] No
Ans : B
108. How many no of columns can a RECORD GROUP have?
a] 10
b] 20
c] 50
d] None of the above
Ans D
109. Oracle precompiler translates the EMBEDDED SQL statemens into
a] Oracle FORMS
b] Oracle REPORTS
c] Oracle LIBRARY
d] None of the above
Ans : D
110. Kind of COMMENT statements placed within SQL statements?
a] Asterisk(*) in column?
b] ANSI SQL style statements(...)
c] C-Style comments (/*......*/)
d] All the above
Ans : D
111. What is the appropriate destination type to send the output to a printer?
a] Screen
b] Previewer
c] Either of the above
d] None of the above
Ans : D
112. What is TERM?
a] TERM is the terminal definition file that describes the terminal from which you are using R20RUN ( Reports run time )
b] TERM is the terminal definition file that describes the terminal from which you are using R20DES ( Reports designer )
c] There is no Parameter called TERM in Reports 2.0
d] None of the above
Ans : A
113. If the maximum records retrieved property of a query is set to 10,then a summary value will be calculated
a] Only for 10 records
b] For all the records retrieved
c] For all therecords in the referenced table
d] None of the above
Ans : A
114. With which function of a summary item in the COMPUTE AT option
required?
a] Sum
b] Standard deviation
c] Variance
d] % of Total function
Ans : D
115. For a field in a repeating frame, can the source come from a column which does not exist in the datagroup which forms the base of the frame?
a] Yes
b] No
Ans : A
116. What are the different file extensions that are created by Oracle Reports?
a] .RDF file & .RPX file
b] .RDX file & .RDF file
c] .REP file & .RDF file
d] None of the above
Ans : C
117. Is it possible to Disable the Parameter form while running the report?
a] Yes
b] No
Ans : A
118.What are the SQL clauses supported in the link property sheet?
a] WHERE & START WITH
b] WHERE & HAVING
c} START WITH & HAVING
d] WHERE, START WITH & HAVING
Ans : D
119. What are the types of Calculated columns available?
a] Summary, Place holder & Procedure column
b] Summary, Procedure & Formula columns
c] Procedure, Formula & Place holder columns
d] Summary, Formula & Place holder columns
Ans.: D
120. If two groups are not linked in the data model editor, what is the hierarchy between them?
a] There is no hierarchy between unlinked groups
b] The group that is right ranks higher than the group that is to the left
c] The group that is above or leftmost ranks higher than the group that is to right or below it
d] None of the above
Ans : C
121. Sequence of events takes place while starting a Database is
a] Database opened, File mounted, Instance started
b] Instance started, Database mounted & Database opened
c] Database opened, Instance started & file mounted
d] Files mounted, Instance started & Database opened
Ans : B
122. SYSTEM TABLESPACE can be made off-line
a] Yes
b] No
Ans : B
123. ENQUEUE_RESOURCES parameter information is derived from
a] PROCESS or DDL_LOCKS & DML_LOCKS
b] LOG BUFFER
c] DB_BLOCK_SIZE
d] DB_BLOCK_BUFFERS
Ans : A
124. SMON process is used to write into LOG files
a] TRUE
b] FALSE
Ans : B
125. EXP command is used
a] To take Backup of the Oracle Database
b] To import data from the exported dump file
c] To create Rollback segments
d] None of the above
Ans : A
126. SNAPSHOTS cannot be refreshed automatically
a] TRUE
b] FALSE
Ans : B
127. The User can set Archive file name formats
a] TRUE
b] FALSE
Ans : A
128. The following parameters are optional in init.ora parameter file DB_BLOCK_SIZE, PROCESS
a} TRUE
b] FALSE
Ans : B
129. NOARCHIEVELOG parameter is used to enable the database in Archieve
mode
a] TRUE
b] FALSE
Ans : B
130. Constraints cannot be exported through Export command?
a] TRUE
b] FALSE
Ans : B
131. It is very difficult to grant and manage common priveleges needed by
different groups of
database users using roles
a] TRUE
b] FALSE
Ans : B
132. The status of the Rollback segment can be viewed through
a] DBA_SEGMENTS
b] DBA_ROLES
c] DBA_FREE_SPACES
d] DBA_ROLLBACK_SEG
Ans : D
133. Explicitly we can assign transaction to a rollback segment
a] TRUE
B] FALSE
Ans : A
134. What file is read by ODBC to load drivers?
a] ODBC.INI
b] ODBC.DLL
c] ODBCDRV.INI
d] None of the above
Ans : A